Annotation of db/baza/rusvuz17.txt, revision 1.1

1.1     ! rubashki    1: Чемпионат:
        !             2: Открытый чемпионат вузов России по ЧГК. 2016/17
        !             3: 
        !             4: Дата:
        !             5: 00-000-2016
        !             6: 
        !             7: Тур:
        !             8: Первый игровой день. 1 тур
        !             9: 
        !            10: Дата:
        !            11: 30-Sep-2016
        !            12: 
        !            13: Редактор:
        !            14: Илья Иванов (Пермь)
        !            15: 
        !            16: Инфо:
        !            17: Редактор благодарит за помощь в подготовке пакета Виктора Абдураманова
        !            18: (Березники), Арсэна Атнагулова (Уфа), Олега Белослудцева, Константина
        !            19: Костенко, Вадима Опутина, Александра Пономарёва, Александра Шишкина,
        !            20: Юлию Шучалову и Дениса Яруллина (все - Пермь), Александра Быкова и Ирину
        !            21: Пинегину (оба - Киров), Андрея Ефремова (Могилев), Андрея Кокуленко
        !            22: (Омск), Александра Мудрого (Черновцы), Константина Науменко (Киев),
        !            23: Ксению Рудич (Москва), Константина Сахарова (Ивантеевка), Ульяну
        !            24: Фабричнину (Санкт-Петербург), а также Дмитрия Башука и команду "От Винта
        !            25: - Братья по фазе" (Харьков).
        !            26: 
        !            27: Вопрос 1:
        !            28: В пражском ресторане "Vytopna" [вЫтопна] процесс доставки пива
        !            29: механизирован: клиенты получают свое пиво, когда происходит ЭТО. Кто
        !            30: создал "ЭТО" в конце XIX века?
        !            31: 
        !            32: Ответ:
        !            33: [Братья] Люмьер.
        !            34: 
        !            35: Комментарий:
        !            36: За доставку пива в ресторане отвечает миниатюрная железная дорога -
        !            37: поезд с бокалом подъезжает к посетителю, и тот может забрать стакан.
        !            38: "Прибытие поезда" - знаменитый фильм братьев Люмьер.
        !            39: 
        !            40: Источник:
        !            41:    1. http://luxlux.net/prazhskiy-restoran-vytopna-poezd-vmesto-ofitsianta-35437/
        !            42:    2. http://ru.wikipedia.org/wiki/Прибытие_поезда_на_вокзал_Ла-Сьота
        !            43: 
        !            44: Автор:
        !            45: Игорь Демьянцев (Гомель)
        !            46: 
        !            47: Вопрос 2:
        !            48: В американском комедийном мультсериале сотрудники одной организации
        !            49: используют фургон, предлагающий свежие буррито. Назовите эту
        !            50: организацию.
        !            51: 
        !            52: Ответ:
        !            53: ФБР.
        !            54: 
        !            55: Зачет:
        !            56: FBI; Федеральное Бюро Расследований; Federal Bureau of Investigation.
        !            57: 
        !            58: Комментарий:
        !            59: Спецслужбы часто используют фургоны для слежки. В мультсериале агенты
        !            60: ФБР (по-английски - FBI) умудрились использовать фургон с надписью
        !            61: "Fresh Burritos Instantly" [фреш буррИтос Инстантли] ("Свежие буррито
        !            62: немедленно"), сокращенно - FBI.
        !            63: 
        !            64: Источник:
        !            65: Мультсериал "Симпсоны", s25e01.
        !            66: 
        !            67: Автор:
        !            68: Илья Иванов (Пермь)
        !            69: 
        !            70: Вопрос 3:
        !            71: В 1894 году астроном Эндрю Дуглас предложил построить обсерваторию в
        !            72: городке Флагстафф в Аризоне. Рассказывают, что в конце жизни Дуглас
        !            73: признался, что во Флагстаффе его интересовали только луны. В одном из
        !            74: слов предыдущего предложения мы пропустили две буквы. Напишите это слово
        !            75: в исходном варианте.
        !            76: 
        !            77: Ответ:
        !            78: Салуны.
        !            79: 
        !            80: Комментарий:
        !            81: Территория славилась своими салунами - Аризона была частью дикого
        !            82: Запада, а дело было в конце XIX века. Вот такой вот "научный" критерий
        !            83: выбора для обсерватории.
        !            84: 
        !            85: Источник:
        !            86:    1. http://ru.wikipedia.org/wiki/Обсерватория_Лоуэлла
        !            87:    2. http://www.peoples.ru/science/astronomy/andrew_ellicott_douglas/
        !            88: 
        !            89: Автор:
        !            90: Илья Иванов (Пермь)
        !            91: 
        !            92: Вопрос 4:
        !            93: По несколько парадоксальному утверждению комментатора, эксцентричный
        !            94: Фабьен Бартез иногда устраивал в воротах ЕЕ. Назовите ЕЕ словом с
        !            95: удвоенной согласной.
        !            96: 
        !            97: Ответ:
        !            98: Буффонада.
        !            99: 
        !           100: Комментарий:
        !           101: Бартез в воротах постоянно "чудил". Джанлуиджи Буффон - другой
        !           102: легендарный футбольный голкипер.
        !           103: 
        !           104: Источник:
        !           105: Трансляция финала чемпионата мира по футболу 1998 года Франция -
        !           106: Бразилия на телеканале "Матч-ТВ", 08.07.2016 г.
        !           107: 
        !           108: Автор:
        !           109: Илья Иванов (Пермь)
        !           110: 
        !           111: Вопрос 5:
        !           112: [Ведущему: обязательно произнести название игры полностью!]
        !           113:    В начале игры "Шерлок Холмс: Преступления и наказания" главный герой,
        !           114: стреляя по вазам, приветствует инспектора Лестрейда, чем изрядно
        !           115: удивляет последнего. Какие три слова мы пропустили в этом вопросе?
        !           116: 
        !           117: Ответ:
        !           118: С завязанными глазами.
        !           119: 
        !           120: Зачет:
        !           121: С закрытыми глазами; другие синонимичные ответы из трех слов.
        !           122: 
        !           123: Комментарий:
        !           124: Несмотря на завязанные глаза, Холмс определяет, что в комнату вошел
        !           125: именно инспектор Лестрейд, - главным образом, благодаря бряцанью
        !           126: наручников, звуку шагов и тому, что "скрипучая" ступенька не скрипнула
        !           127: (значит, гость бывает в доме часто).
        !           128: 
        !           129: Источник:
        !           130: Игра "Sherlock Holmes: Crimes and Punishments".
        !           131: 
        !           132: Автор:
        !           133: Илья Иванов (Пермь)
        !           134: 
        !           135: Вопрос 6:
        !           136: Герой фильма "Космос как предчувствие" знакомится в поезде с молодым
        !           137: летчиком Юрием. Когда тот направляется к выходу, наблюдательный герой
        !           138: окликает его и указывает на один из НИХ. Назовите ИХ.
        !           139: 
        !           140: Ответ:
        !           141: Шнурки.
        !           142: 
        !           143: Комментарий:
        !           144: Создатели фильма явно решили сделать отсылку к известной истории, когда
        !           145: у Гагарина, шедшего по аэродрому Внуково к правительственной трибуне,
        !           146: развязался шнурок. Впрочем, существует версия, что это были подтяжки для
        !           147: носков - однако в фильме, конечно, фигурировали именно шнурки.
        !           148: 
        !           149: Источник:
        !           150:    1. Х/ф "Космос как предчувствие" (2005), реж. Алексей Учитель.
        !           151:    2. http://www.retroportal.ru/gagarin_14_04_1961.shtml
        !           152: 
        !           153: Автор:
        !           154: Илья Иванов (Пермь)
        !           155: 
        !           156: Вопрос 7:
        !           157: Герой французской комедии любезно отдает молодой немке солонку и
        !           158: перечницу. Недовольная жена язвительно спрашивает, почему же он не
        !           159: решился предложить девушке арию. В одном из слов предыдущего предложения
        !           160: мы пропустили шесть букв. Напишите это слово в исходном варианте.
        !           161: 
        !           162: Ответ:
        !           163: Лотарингию.
        !           164: 
        !           165: Комментарий:
        !           166: Лотарингия - знаменитая провинция, которая (вместе с Эльзасом) долгое
        !           167: время была причиной ожесточенных сражений и споров между Францией и
        !           168: Германией.
        !           169: 
        !           170: Источник:
        !           171: Х/ф "Каникулы маленького Николя" (2014), реж. Лоран Тирар.
        !           172: 
        !           173: Автор:
        !           174: Илья Иванов (Пермь)
        !           175: 
        !           176: Вопрос 8:
        !           177: В уже упомянутой нами игре на стене в комнате Шерлока Холмса можно
        !           178: увидеть ИКС, написанный в 1872 году. Назовите ИКС двумя словами.
        !           179: 
        !           180: Ответ:
        !           181: Портрет Достоевского.
        !           182: 
        !           183: Комментарий:
        !           184: Игра называется "Шерлок Холмс: Преступления и наказания", что является
        !           185: отсылкой к знаменитому роману Федора Михайловича.
        !           186: 
        !           187: Источник:
        !           188:    1. Игра "Sherlock Holmes: Crimes and Punishments".
        !           189:    2. https://www.wikiart.org/ru/vasiliy-perov/portret-pisatelya-fedora-mikhaylovicha-dostoevskogo-1872
        !           190: 
        !           191: Автор:
        !           192: Илья Иванов (Пермь)
        !           193: 
        !           194: Вопрос 9:
        !           195: На карикатуре Жана Эффеля Бог, прогуливающийся среди хаоса, замечает,
        !           196: что мир - это ИКС. Одним из лучших ИКСОВ 2015 года был признан
        !           197: "нипричёмыш". Какое слово греческого происхождения мы заменили ИКСОМ?
        !           198: 
        !           199: Ответ:
        !           200: Неологизм.
        !           201: 
        !           202: Комментарий:
        !           203: На карикатуре, посвященной сотворению мира, Бог достаточно справедливо
        !           204: называет создаваемый им мир неологизмом. "Нипричёмыш" - скучный, тихий
        !           205: человек, всегда держащийся подальше от любых волнений, - был признан
        !           206: Экспертным советом при Центре творческого развития русского языка одним
        !           207: из лучших неологизмов 2015 года.
        !           208: 
        !           209: Источник:
        !           210:    1. Ж. Эффель. Сотворение мира. - М.: Изобразительное искусство, 1989.
        !           211:    2. https://snob.ru/profile/27356/blog/101993
        !           212: 
        !           213: Автор:
        !           214: Виктор Абдураманов (Березники)
        !           215: 
        !           216: Вопрос 10:
        !           217: По анекдотической версии Александра Экстера, жена однажды увидела
        !           218: картину мужа из цикла работ о женской бане и пришла в ярость. Какое
        !           219: произведение, согласно этой версии, было создано в результате?
        !           220: 
        !           221: Ответ:
        !           222: "Черный квадрат".
        !           223: 
        !           224: Комментарий:
        !           225: И в ярости замазала полотно черной краской. Так Малевич и придумал свой
        !           226: "Черный квадрат" (во что, конечно, верится с трудом).
        !           227: 
        !           228: Источник:
        !           229: http://www.vokrugsveta.ru/article/205155/
        !           230: 
        !           231: Автор:
        !           232: Илья Иванов (Пермь)
        !           233: 
        !           234: Вопрос 11:
        !           235: Одна из шведских церквей недавно вывесила объявление с напоминанием, что
        !           236: по всё еще действующему закону 1280 года никто не может быть взят в плен
        !           237: в церкви. Отдельно подчеркнуто, что под защиту закона подпадают и ОНИ.
        !           238: Назовите ИХ одним словом.
        !           239: 
        !           240: Ответ:
        !           241: Покемоны.
        !           242: 
        !           243: Комментарий:
        !           244: Судя по всему, любители поохотиться на покемонов делали это и в церкви.
        !           245: 
        !           246: Источник:
        !           247: http://www.diary.ru/~Logovojuni/p209907950.htm
        !           248: 
        !           249: Автор:
        !           250: Константин Костенко (Гремячинск - Пермь)
        !           251: 
        !           252: Вопрос 12:
        !           253: Опера ДжАкомо Пуччини "ТОска" заканчивается прыжком главной героини с
        !           254: высокой башни. По легенде, конец одной из постановок "ТОски" привел
        !           255: публику в восторг. Причиной этого стало использование ЕГО. Назовите ЕГО
        !           256: одним словом.
        !           257: 
        !           258: Ответ:
        !           259: Батут.
        !           260: 
        !           261: Комментарий:
        !           262: Исполнительница главной роли спрыгнула из декорационной башни, однако
        !           263: вместо привычных матрасов кто-то додумался установить батут. В итоге
        !           264: певица три или четыре раза подпрыгнула на нем, что не осталось
        !           265: незамеченным публикой.
        !           266: 
        !           267: Источник:
        !           268: Э. Ланди. Тайная жизнь великих композиторов.
        !           269: http://www.flibusta.is/b/390922/read
        !           270: 
        !           271: Автор:
        !           272: Илья Иванов (Пермь)
        !           273: 
        !           274: Тур:
        !           275: Первый игровой день. 2 тур
        !           276: 
        !           277: Дата:
        !           278: 30-Sep-2016
        !           279: 
        !           280: Редактор:
        !           281: Андрей Ефремов (Могилев)
        !           282: 
        !           283: Инфо:
        !           284: Редактор благодарит за помощь в тестировании Валерия Семёнова, Сергея
        !           285: Дубелевича и команду "Солянка" (все - Минск), команду "От винта - Братья
        !           286: по фазе" (Харьков) и Лидию Иоффе (Хайфа).
        !           287: 
        !           288: Вопрос 1:
        !           289: В научно-популярном фильме параллельная эволюция вирусов и человеческого
        !           290: организма сравнивается с НЕЙ. Назовите ЕЕ двумя словами, начинающимися
        !           291: на соседние буквы алфавита.
        !           292: 
        !           293: Ответ:
        !           294: Гонка вооружений.
        !           295: 
        !           296: Комментарий:
        !           297: Вирусы и человеческие клетки на протяжении тысячелетий вынуждены были
        !           298: изобретать всё новые и новые механизмы атаки и защиты, обмана и его
        !           299: распознавания.
        !           300: 
        !           301: Источник:
        !           302: Эфир телеканала "Наука 2.0", сериал "Тайная вселенная", серия
        !           303: "Путешествие внутрь клетки", эфир от 11.02.2015 г.
        !           304: 
        !           305: Автор:
        !           306: Андрей Ефремов (Могилев)
        !           307: 
        !           308: Вопрос 2:
        !           309: Автор исторического романа сравнил ЕЕ с заточённой в монастырь царицей,
        !           310: которую ветреный и капризный властелин променял на холодную,
        !           311: змеиноглазую разлучницу. Назовите ЕЕ одним словом.
        !           312: 
        !           313: Ответ:
        !           314: Москва.
        !           315: 
        !           316: Комментарий:
        !           317: Как известно, в XVIII веке Петр I перенес столицу в Санкт-Петербург.
        !           318: 
        !           319: Источник:
        !           320: Б. Акунин. Любовница смерти. http://www.flibusta.is/b/500333/read
        !           321: 
        !           322: Автор:
        !           323: Андрей Ефремов (Могилев)
        !           324: 
        !           325: Вопрос 3:
        !           326: В статье об ИКСЕ упоминаются коронарные артерии. В произведении XIV века
        !           327: обитатели ИКСА стараются прикрыть один бок другим. Назовите ИКС двумя
        !           328: словами.
        !           329: 
        !           330: Ответ:
        !           331: Третий круг.
        !           332: 
        !           333: Комментарий:
        !           334: Третий круг кровообращения обслуживает само сердце. Во втором
        !           335: предложении речь идет о третьем круге ада в описании Данте, в который
        !           336: помещены чревоугодники, стыдящиеся своей полноты. Вопрос в туре идет
        !           337: третьим - это дополнительная подсказка.
        !           338: 
        !           339: Источник:
        !           340:    1. http://www.science-education.ru/34-1342
        !           341:    2. https://otvet.mail.ru/question/47811228
        !           342: 
        !           343: Автор:
        !           344: Андрей Ефремов (Могилев)
        !           345: 
        !           346: Вопрос 4:
        !           347: На карикатуре, действие которой происходит весной, ПЕРВЫЕ испуганно
        !           348: смотрят на появляющиеся из земли ВТОРЫЕ. Какие однокоренные слова мы
        !           349: обозначили как "ПЕРВЫЕ" и "ВТОРЫЕ"?
        !           350: 
        !           351: Ответ:
        !           352: Снеговики, подснежники.
        !           353: 
        !           354: Источник:
        !           355: http://pikabu.ru/story/predvestniki_smerti_1944655
        !           356: 
        !           357: Автор:
        !           358: Андрей Ефремов (Могилев)
        !           359: 
        !           360: Вопрос 5:
        !           361: В одном из стихотворений Владимир Маяковский удивляется: неужели "зуб
        !           362: революций ступился о НИХ"? Какая денежная единица получила свое название
        !           363: от НЕЕ?
        !           364: 
        !           365: Ответ:
        !           366: Крона.
        !           367: 
        !           368: Комментарий:
        !           369: ОНА - это корона. Как известно, прокатившаяся по Европе волна революций
        !           370: способствовала падению не одной монархии.
        !           371: 
        !           372: Источник:
        !           373:    1. В.В. Маяковский. Радоваться рано.
        !           374: http://www.feb-web.ru/feb/mayakovsky/texts/ms0/ms2/ms2-016-.htm
        !           375:    2. http://ru.wikipedia.org/wiki/Крона_(денежная_единица)
        !           376: 
        !           377: Автор:
        !           378: Андрей Ефремов (Могилев)
        !           379: 
        !           380: Вопрос 6:
        !           381: В вопросе словом "ИКС" заменено другое слово.
        !           382:    Однажды в компьютерной игре "Герои меча и магии - 5" автору вопроса
        !           383: встретился ИКС дьяволят. Назовите ИКС, созданный в 1831 году королевским
        !           384: приказом.
        !           385: 
        !           386: Ответ:
        !           387: [Французский] Иностранный [легион].
        !           388: 
        !           389: Комментарий:
        !           390: Словосочетание "легион дьяволят" отсылает к цитате из Евангелия "Имя мне
        !           391: - легион". Иностранный легион был создан по приказу Луи-Филиппа I, а
        !           392: Почетный легион - еще при Наполеоне I.
        !           393: 
        !           394: Источник:
        !           395:    1. ЛОАВ (пользовательская карта "Живее всех живых").
        !           396:    2. http://ru.wikipedia.org/wiki/Имя_мне_%E2%80%94_легион
        !           397:    3. http://ru.wikipedia.org/wiki/Французский_Иностранный_легион
        !           398: 
        !           399: Автор:
        !           400: Андрей Ефремов (Могилев)
        !           401: 
        !           402: Вопрос 7:
        !           403: В сентябре 1957 года сразу после аварии на химкомбинате "Маяк" в
        !           404: челябинских газетах появились сообщения о НЕМ. ОНО бывает двух видов,
        !           405: которые на самом деле вовсе не симметричны. Назовите ЕГО двумя словами,
        !           406: начинающимися на разные буквы.
        !           407: 
        !           408: Ответ:
        !           409: Полярное сияние.
        !           410: 
        !           411: Комментарий:
        !           412: Советские газеты пытались дезинформировать население, выдав техногенную
        !           413: катастрофу за природное явление. До недавнего времени считалось, что
        !           414: полярные сияния в северном и южном полушарии являются симметричными.
        !           415: Однако одновременное наблюдение полярного сияния в мае 2001 года из
        !           416: космоса со стороны северного и южного полюсов показало, что северное и
        !           417: южное сияния существенно отличаются друг от друга.
        !           418: 
        !           419: Источник:
        !           420:    1. http://ru.wikipedia.org/wiki/Кыштымская_авария
        !           421:    2. http://ru.wikipedia.org/wiki/Полярное_сияние
        !           422: 
        !           423: Автор:
        !           424: Андрей Ефремов (Могилев)
        !           425: 
        !           426: Вопрос 8:
        !           427: Описывая шумную толпу в лондонском порту, Филип Депуа пишет, что
        !           428: невидимая ОНА поднималась до самого неба. Назовите ЕЕ двумя словами.
        !           429: 
        !           430: Ответ:
        !           431: Вавилонская башня.
        !           432: 
        !           433: Комментарий:
        !           434: Писатель упоминает "разноязыкий гомон". Слово "толпа" в вопросе -
        !           435: подсказка.
        !           436: 
        !           437: Источник:
        !           438: Ф. Депуа. Тайна короля Якова. http://www.flibusta.is/b/254925/read
        !           439: 
        !           440: Автор:
        !           441: Андрей Ефремов (Могилев)
        !           442: 
        !           443: Вопрос 9:
        !           444: При создании ИХ в резиденции сёгуна гвозди располагали в виде
        !           445: перевернутой буквы V [ви] для имитации щебета различных птиц, по
        !           446: которому можно было достаточно точно определить местонахождение
        !           447: посетителей. Назовите ИХ двумя словами, начинающимися на одну и ту же
        !           448: букву.
        !           449: 
        !           450: Ответ:
        !           451: Поющие полы.
        !           452: 
        !           453: Зачет:
        !           454: Птичьи полы.
        !           455: 
        !           456: Комментарий:
        !           457: Поющие (соловьиные) полы были созданы в замке сёгунов в Киото, в первую
        !           458: очередь, для того чтобы враги не могли пробраться туда незамеченными. В
        !           459: зависимости от того, голос какой именно птицы был слышен, можно было
        !           460: точно определить местонахождение проникнувшего в замок.
        !           461: 
        !           462: Источник:
        !           463: https://historyporn.dirty.ru/poiushchie-poly-na-strazhe-siogunov-540719/
        !           464: 
        !           465: Автор:
        !           466: Андрей Ефремов (Могилев)
        !           467: 
        !           468: Вопрос 10:
        !           469: Престарелый герой рассказа Стивена Кинга сравнивает свое тело с
        !           470: разрушенным ИКСОМ. Самый высокий ИКС был создан в Майами. Назовите ИКС.
        !           471: 
        !           472: Ответ:
        !           473: Замок из песка.
        !           474: 
        !           475: Зачет:
        !           476: Синонимичные ответы.
        !           477: 
        !           478: Комментарий:
        !           479: Известно выражение "песок сыплется". Город Майами известен своими
        !           480: пляжами. Кстати, предыдущий вопрос тоже был о зАмке.
        !           481: 
        !           482: Источник:
        !           483:    1. С. Кинг. Человек в черном костюме.
        !           484: http://www.flibusta.is/b/182301/read
        !           485:    2. http://lenta.ru/news/2015/10/27/zamok/
        !           486: 
        !           487: Автор:
        !           488: Андрей Ефремов (Могилев)
        !           489: 
        !           490: Вопрос 11:
        !           491: Блиц.
        !           492:    На одном специализированном обучающем сайте есть цикл детских
        !           493: стихотворений.
        !           494:    1. В одном из стихотворений ОНА "словно подлодка, стреляет прямой
        !           495: наводкой". Назовите ЕЕ одним словом.
        !           496:    2. В одном из стихотворений ОН предпочитает рукопашную. Назовите ЕГО
        !           497: одним словом.
        !           498:    3. В одном из стихотворений ОНА решает, кем стать. Назовите ЕЕ одним
        !           499: словом.
        !           500: 
        !           501: Ответ:
        !           502:    1. Ладья.
        !           503:    2. Король.
        !           504:    3. Пешка.
        !           505: 
        !           506: Комментарий:
        !           507: В шахматах ладья контролирует вертикаль и горизонталь, король бьет
        !           508: только соседние клетки, а пешка, дойдя до последней горизонтали, может,
        !           509: в зависимости от решения игрока, превратиться в другую фигуру.
        !           510: 
        !           511: Источник:
        !           512:    1. http://fenix64.com/stix-o-lade/
        !           513:    2. http://fenix64.com/stix-pro-korolya-2/
        !           514:    3. http://fenix64.com/stix-pro-peshku/
        !           515: 
        !           516: Автор:
        !           517: Андрей Ефремов (Могилев)
        !           518: 
        !           519: Вопрос 12:
        !           520: Решительно настроенный президент США Уильям Тафт считал, что "ЭТО" может
        !           521: успокоить забастовку рабочих. Обычно за один сеанс ЭТОГО устанавливают
        !           522: от четырех до двенадцати... Кого?
        !           523: 
        !           524: Ответ:
        !           525: Пиявок.
        !           526: 
        !           527: Комментарий:
        !           528: Речь идет о кровопускании. Тафт ратовал за применение силы в отношении
        !           529: бастующих.
        !           530: 
        !           531: Источник:
        !           532:    1. http://ru.wikipedia.org/wiki/Тафт,_Уильям_Говард
        !           533:    2. http://ru.wikipedia.org/wiki/Кровопускание
        !           534: 
        !           535: Автор:
        !           536: Андрей Ефремов (Могилев)
        !           537: 
        !           538: Тур:
        !           539: Первый игровой день. 3 тур
        !           540: 
        !           541: Дата:
        !           542: 30-Sep-2016
        !           543: 
        !           544: Редактор:
        !           545: Дмитрий Башук (Харьков)
        !           546: 
        !           547: Инфо:
        !           548: Редактор благодарит за помощь в подготовке вопросов команду "От винта -
        !           549: Братья по фазе" (Харьков).
        !           550: 
        !           551: Вопрос 1:
        !           552: Шутники утверждают, что для грузчиков характерно раздельное питание.
        !           553: Когда они разгружают картошку, то едят картошку, когда разгружают сыр -
        !           554: едят сыр. А когда разгружают кирпич, то это - ОН. Назовите ЕГО двумя
        !           555: словами.
        !           556: 
        !           557: Ответ:
        !           558: Разгрузочный день.
        !           559: 
        !           560: Комментарий:
        !           561: "Разгрузочный день", как и "раздельное питание", - устойчивое понятие в
        !           562: диетологии.
        !           563: 
        !           564: Источник:
        !           565: "Арт-Мозаика", 2016, N 34.
        !           566: 
        !           567: Автор:
        !           568: Дмитрий Башук (Харьков)
        !           569: 
        !           570: Вопрос 2:
        !           571: Одну из актрис Фаина Раневская причислила к отряду молеобразных и
        !           572: объяснила это тем, что у той все мысли только об АЛЬФАХ. Стихотворение
        !           573: Семена Кирсанова об АЛЬФЕ заканчивается словами "... чтоб не мог мороз
        !           574: ущипнуть тебя". Назовите АЛЬФУ.
        !           575: 
        !           576: Ответ:
        !           577: Шуба.
        !           578: 
        !           579: Источник:
        !           580:    1. Ф.Г. Раневская. Мудрые остроты Раневской.
        !           581: http://www.flibusta.is/b/380824/read
        !           582:    2. С.И. Кирсанов. Нащот шубы.
        !           583: http://www.flibusta.is/b/343461/read#t21
        !           584: 
        !           585: Автор:
        !           586: Дмитрий Башук (Харьков)
        !           587: 
        !           588: Вопрос 3:
        !           589: Для десятилетнего Нила Сети, исполнившего роль Маугли в недавнем фильме
        !           590: "Книга джунглей", костюмеры изготовили двадцать вариантов ИХ, которые
        !           591: меняли в зависимости от эпизода. Назовите ИХ двумя словами.
        !           592: 
        !           593: Ответ:
        !           594: Набедренные повязки.
        !           595: 
        !           596: Комментарий:
        !           597: Других костюмов Маугли и не надо. :-)
        !           598: 
        !           599: Источник:
        !           600: "Телескоп", 2016, N 16.
        !           601: 
        !           602: Автор:
        !           603: Дмитрий Башук (Харьков)
        !           604: 
        !           605: Вопрос 4:
        !           606: У основания кафедры венского собора Святого Стефана есть скульптурное
        !           607: изображение человека с циркулем в руке. Назовите это изображение словом,
        !           608: происходящим от древнегреческого и французского слов.
        !           609: 
        !           610: Ответ:
        !           611: Автопортрет.
        !           612: 
        !           613: Комментарий:
        !           614: (pic: 20160953.jpg)
        !           615:    Создатель кафедры каменотёс Антон Пильграм изобразил себя взирающим
        !           616: на дело своих рук; слово "автопортрет" происходит от древнегреческого
        !           617: "autos" и французского "portrait".
        !           618: 
        !           619: Источник:
        !           620:    1. Е.Н. Грицак. Вена. http://www.flibusta.is/b/116707/read
        !           621:    2. http://ru.wiktionary.org/wiki/автопортрет
        !           622: 
        !           623: Автор:
        !           624: Дмитрий Башук (Харьков)
        !           625: 
        !           626: Вопрос 5:
        !           627: [Ведущему: при чтении явно указать запятую после первого пропуска.]
        !           628:    Опальный поэт из книги Артуро Переса-Реверте в одном из эпизодов так
        !           629: каламбурит о себе: "Дела идут [пропуск], дела [пропуск] дышат...".
        !           630: Заполните пропуски - двумя словами каждый.
        !           631: 
        !           632: Ответ:
        !           633: "... на лад...", "... на ладан...".
        !           634: 
        !           635: Комментарий:
        !           636: Такой вот грустный каламбур.
        !           637: 
        !           638: Источник:
        !           639: А. Перес-Реверте. Капитан Алатристе.
        !           640: http://www.flibusta.is/b/456025/read
        !           641: 
        !           642: Автор:
        !           643: Дмитрий Башук (Харьков)
        !           644: 
        !           645: Вопрос 6:
        !           646: Материал журнала "Вокруг света" под заголовком "Личное влияние" посвящен
        !           647: ИМ. Кстати, одна из ИХ разновидностей называлась словом, однокоренным к
        !           648: слову из приведенного заголовка. Назовите ИХ.
        !           649: 
        !           650: Ответ:
        !           651: Маски.
        !           652: 
        !           653: Комментарий:
        !           654: Металлическая маска под названием "личина" была частью военного шлема.
        !           655: 
        !           656: Источник:
        !           657:    1. "Вокруг света", 2016, N 7.
        !           658:    2. http://ru.wikipedia.org/wiki/Личина
        !           659: 
        !           660: Автор:
        !           661: Дмитрий Башук (Харьков)
        !           662: 
        !           663: Вопрос 7:
        !           664: Площадь ЕЕ составляет около 4152 квадратных километров и с каждым годом
        !           665: увеличивается. В 1991 году румынская часть, составляющая примерно 83%
        !           666: общей площади, была признана ЮНЕСКО объектом Всемирного природного
        !           667: наследия. Назовите ЕЕ двумя словами, которые начинаются на одну и ту же
        !           668: букву.
        !           669: 
        !           670: Ответ:
        !           671: Дельта Дуная.
        !           672: 
        !           673: Комментарий:
        !           674: Остальные 17% принадлежат Украине; с каждым годом береговая линия
        !           675: Черного моря смещается метров на сорок вглубь моря из-за наносов.
        !           676: 
        !           677: Источник:
        !           678: http://ru.wikipedia.org/wiki/Дельта_Дуная
        !           679: 
        !           680: Автор:
        !           681: Александр Лисянский (Харьков)
        !           682: 
        !           683: Вопрос 8:
        !           684: В отличие от математики, в жизни минус на минус не всегда дает плюс.
        !           685: Видимо, поэтому, согласно закону города Лик-Спрингс в штате Индиана,
        !           686: выпуская на улицу черных котов в определенный день, хозяева обязаны
        !           687: надевать на своих питомцев колокольчики. Назовите этот день абсолютно
        !           688: точно.
        !           689: 
        !           690: Ответ:
        !           691: Пятница, 13-е.
        !           692: 
        !           693: Комментарий:
        !           694: Авторы закона, видимо, считают, что если нельзя избежать негативного
        !           695: влияния пятницы, 13-го, то хотя бы черных котов, собирающихся перейти
        !           696: дорогу, горожане в этот день заблаговременно обнаружат по звуку
        !           697: колокольчиков. :-)
        !           698: 
        !           699: Источник:
        !           700: http://www.koshsps.ru/blackcat.php
        !           701: 
        !           702: Автор:
        !           703: Дмитрий Башук (Харьков)
        !           704: 
        !           705: Вопрос 9:
        !           706: Персонаж книги Хольма Ван Зайчика рассуждает: "Невозможно представить
        !           707: себе в просторах Запретного города, скажем, велосипед... это вопиюще
        !           708: несообразно и даже несколько оскорбительно - как если бы, скажем, ИКС
        !           709: вдруг запел...". Чуть больше сорока пяти лет назад так и произошло.
        !           710: Назовите ИКСА.
        !           711: 
        !           712: Ответ:
        !           713: Иисус Христос.
        !           714: 
        !           715: Комментарий:
        !           716: В 1970 году была создана рок-опера "Иисус Христос - суперзвезда".
        !           717: 
        !           718: Источник:
        !           719:    1. Х. Ван Зайчик. Дело Судьи Ди. http://www.flibusta.is/b/73894/read
        !           720:    2. http://ru.wikipedia.org/wiki/Иисус_Христос_%E2%80%94_суперзвезда
        !           721: 
        !           722: Автор:
        !           723: Дмитрий Башук (Харьков)
        !           724: 
        !           725: Вопрос 10:
        !           726: Одной из причин распада этой группы в апреле 1981 года Лоуренс ДжУбер
        !           727: считает то, что ее лидер стал бояться концертов (цитата): "Ведь ему
        !           728: приходилось бы каждые десять минут вздрагивать, ожидая, что какой-нибудь
        !           729: придурок выстрелит в него из пистолета". Назовите эту группу.
        !           730: 
        !           731: Ответ:
        !           732: "Wings".
        !           733: 
        !           734: Комментарий:
        !           735: Джубер - гитарист этой группы. Убийство Джона Леннона, произошедшее 8
        !           736: декабря 1980 года, сильно потрясло Пола Маккартни, создавшего "Wings" в
        !           737: 1971 году, вскоре после распада "Битлз".
        !           738: 
        !           739: Источник:
        !           740: http://ru.wikipedia.org/wiki/Wings
        !           741: 
        !           742: Автор:
        !           743: Дмитрий Башук (Харьков)
        !           744: 
        !           745: Вопрос 11:
        !           746: На западе США колючую проволоку часто использовали в качестве АЛЬФЫ,
        !           747: чтобы не беспокоиться при возможных порывах ветра. Назовите АЛЬФУ двумя
        !           748: словами.
        !           749: 
        !           750: Ответ:
        !           751: Бельевая веревка.
        !           752: 
        !           753: Комментарий:
        !           754: Шипы колючей проволоки легко удерживали сохнущую на ветру одежду.
        !           755: 
        !           756: Источник:
        !           757: http://warspot.ru/2560-shipy-bez-roz
        !           758: 
        !           759: Автор:
        !           760: Максим Евланов (Харьков)
        !           761: 
        !           762: Вопрос 12:
        !           763: "Ты и есть мой дом" - такова, по мнению персонажа Макса Фрая, идеальная
        !           764: ОНА. Премьера "ЕЕ" состоялась 30 декабря 1984 года. Назовите ЕЕ.
        !           765: 
        !           766: Ответ:
        !           767: Формула любви.
        !           768: 
        !           769: Комментарий:
        !           770: Во втором случае речь идет о фильме Марка Захарова.
        !           771: 
        !           772: Источник:
        !           773:    1. М. Фрай. Сказки старого Вильнюса II.
        !           774: http://www.flibusta.is/b/431642/read
        !           775:    2. http://ru.wikipedia.org/wiki/Формула_любви
        !           776: 
        !           777: Автор:
        !           778: Дмитрий Башук (Харьков)
        !           779: 
        !           780: Тур:
        !           781: Второй игровой день. 1 тур
        !           782: 
        !           783: Дата:
        !           784: 11-Nov-2016
        !           785: 
        !           786: Редактор:
        !           787: Артем Матухно (Одесса)
        !           788: 
        !           789: Инфо:
        !           790: Редактор благодарит за помощь и дельные советы во время тестирования:
        !           791: Дениса Гончара, Андрея Дунаева, Ирину Буртненко, Игоря Гайдута, Сергея
        !           792: Гайкова, Елену Григоращенко, Андрея Герасименко, Евгения Криворученко,
        !           793: Алексея Топышева, Андрея Ландера, Влада Гаврилова.
        !           794: 
        !           795: Вопрос 1:
        !           796: Для одной традиции компания "Гиннесс" использует вместо ЭТОГО продукцию
        !           797: собственного производства. В США в годы "сухого закона" вместо ЭТОГО
        !           798: использовали кока-колу. Назовите ЭТО.
        !           799: 
        !           800: Ответ:
        !           801: Шампанское.
        !           802: 
        !           803: Зачет:
        !           804: Бутылка шампанского.
        !           805: 
        !           806: Комментарий:
        !           807: Речь идет о традиции "крещения корабля" с помощью бутылки шампанского.
        !           808: Пивоваренная фирма "Гиннесс" использует при крещении своих судов,
        !           809: предназначенных для перевозки пива, собственное пиво. В добрый путь,
        !           810: друзья!
        !           811: 
        !           812: Источник:
        !           813: http://www.rg-rb.de/index.php?option=com_rg&task=item&id=15792&Itemid=0
        !           814: 
        !           815: Автор:
        !           816: Артем Матухно (Одесса)
        !           817: 
        !           818: Вопрос 2:
        !           819: В 1934 году подготовленная тренером КатрИн Кёртис группа из шестидесяти
        !           820: человек под названием "Современные русалки" с успехом выступила на
        !           821: Всемирной выставке в Чикаго. Какое словосочетание из двух слов впервые в
        !           822: истории использовал диктор НОрман Росс, комментируя это выступление?
        !           823: 
        !           824: Ответ:
        !           825: Синхронное плавание.
        !           826: 
        !           827: Комментарий:
        !           828: Правда, сама Катрин предпочитала термин "ритмическое плавание".
        !           829: 
        !           830: Источник:
        !           831: David Goldblatt. How to Watch the Olympics: An Instant Initiation into
        !           832: Every Sport at Rio-2016.
        !           833: https://books.google.ru/books?id=RqaCCwAAQBAJ&pg=PT366#v=onepage&q&f=false
        !           834: 
        !           835: Автор:
        !           836: Артем Матухно (Одесса)
        !           837: 
        !           838: Вопрос 3:
        !           839: [Ведущему: начинать читать текст вопроса со слова "внимание" - это не
        !           840: ошибка и не опечатка.]
        !           841:    Внимание, вопрос номер два!
        !           842:    Рассказывают, что некоторое время ОН, уроженец Екатеринославской
        !           843: губернии, специально посещал школьные уроки, где, сидя с блокнотом на
        !           844: задних партах, что-то зарисовывал. Назовите ЕГО.
        !           845: 
        !           846: Ответ:
        !           847: [Федор Павлович] Решетников.
        !           848: 
        !           849: Комментарий:
        !           850: Художник посещал уроки в школе перед написанием своей известной картины
        !           851: "Опять двойка". Ведущий намеренно ошибся, чуть не сделав третий вопрос
        !           852: "опять вторым".
        !           853: 
        !           854: Источник:
        !           855: http://www.vokrugsveta.ru/article/252015/
        !           856: 
        !           857: Автор:
        !           858: Артем Матухно (Одесса)
        !           859: 
        !           860: Вопрос 4:
        !           861: Военачальника Николая Муравьёва-Карсского и декабриста Петра Колошина
        !           862: объединяла настолько близкая дружба, что даже своих коней они назвали в
        !           863: честь ИКСОВ. В честь ИКСОВ названы две соседние вершины-четырехтысячники
        !           864: в Альпах. Назовите имена ИКСОВ.
        !           865: 
        !           866: Ответ:
        !           867: Кастор, Поллукс.
        !           868: 
        !           869: Зачет:
        !           870: Кастор, Полидевк.
        !           871: 
        !           872: Комментарий:
        !           873: ИКСЫ - Диоскуры. Муравьёв-Карсский и Колошин были настолько близкими
        !           874: друзьями, что часто называли друг друга братьями. Мифические герои,
        !           875: близнецы Кастор и Поллукс, были известны своей неразлучной дружбой.
        !           876: 
        !           877: Источник:
        !           878:    1. Н.Н. Муравьёв-Карсский. Собственные записки. 1811-1816.
        !           879: http://www.flibusta.is/b/450099/read
        !           880:    2. http://en.wikipedia.org/wiki/Castor_(mountain)
        !           881:    3. http://en.wikipedia.org/wiki/Pollux_(mountain)
        !           882: 
        !           883: Автор:
        !           884: Артем Матухно (Одесса)
        !           885: 
        !           886: Вопрос 5:
        !           887: По словам Юрия Рылёва, автора книги "6000 изобретений XX и XXI веков,
        !           888: изменивших мир", разработка противозачаточных таблеток в середине
        !           889: прошлого века стала залпом "Авроры" для НЕЕ. Назовите ЕЕ точно.
        !           890: 
        !           891: Ответ:
        !           892: Сексуальная революция.
        !           893: 
        !           894: Источник:
        !           895: Ю.И. Рылёв. 6000 изобретений XX и XXI веков, изменившие мир.
        !           896: http://www.flibusta.is/b/377039/read
        !           897: 
        !           898: Автор:
        !           899: Артем Матухно (Одесса)
        !           900: 
        !           901: Вопрос 6:
        !           902: Название одного магазина, предоставляющего услуги по ремонту смартфонов,
        !           903: представляет собой английское слово-неологизм. Какой фрукт присутствует
        !           904: на логотипе этого магазина?
        !           905: 
        !           906: Ответ:
        !           907: Груша.
        !           908: 
        !           909: Зачет:
        !           910: Pear.
        !           911: 
        !           912: Комментарий:
        !           913: Магазин называется "re:Pear" [ре пЭа], что созвучно английскому глаголу
        !           914: "repair" [репЭа] - чинить, ремонтировать.
        !           915: 
        !           916: Источник:
        !           917: Личные наблюдения автора вопроса в Стокгольме.
        !           918: 
        !           919: Автор:
        !           920: Артем Матухно (Одесса)
        !           921: 
        !           922: Вопрос 7:
        !           923: Назовите многократного олимпийского чемпиона, чья фамилия, по одной из
        !           924: версий, могла быть образована от прозвища, которым когда-то называли
        !           925: слишком прилипчивого зануду.
        !           926: 
        !           927: Ответ:
        !           928: [Евгений Викторович] Плющенко.
        !           929: 
        !           930: Комментарий:
        !           931: Прилипчивый, словно плющ. Евгений Плющенко - многократный олимпийский
        !           932: чемпион в фигурном катании.
        !           933: 
        !           934: Источник:
        !           935:    1. Т.Ф. Ведина. Энциклопедия русских фамилий. Тайны происхождения и
        !           936: значения. http://www.flibusta.is/b/154189/read
        !           937:    2. http://ru.wikipedia.org/wiki/Плющенко,_Евгений_Викторович
        !           938: 
        !           939: Автор:
        !           940: Артем Матухно (Одесса)
        !           941: 
        !           942: Вопрос 8:
        !           943: [Ведущему: при чтении выделить интонацией словосочетание "не он".]
        !           944:    Неизвестно, нравилось ли это произведение издателю, но в изначальном
        !           945: авторском варианте был не ОН, а "ключ". В каком южном городе ОН
        !           946: находится до сих пор?
        !           947: 
        !           948: Ответ:
        !           949: Бахчисарай.
        !           950: 
        !           951: Комментарий:
        !           952: Речь идет о Бахчисарайском фонтане. Пушкин изначально назвал свою поэму
        !           953: "Бахчисарайский ключ", а в "фонтан" она превратилась по воле издателя -
        !           954: князя Вяземского. Возможно, он считал, что авторский вариант был, как
        !           955: говорится, "не фонтан".
        !           956: 
        !           957: Источник:
        !           958: http://www.vokrugsveta.ru/vs/article/8482/
        !           959: 
        !           960: Автор:
        !           961: Артем Матухно (Одесса)
        !           962: 
        !           963: Вопрос 9:
        !           964: Согласно "Словарю морского жаргона", АЛЬФА - это прозвище
        !           965: "привилегированной женской особы, работающей в команде судна". АЛЬФОЙ
        !           966: была персонаж серии детских книг, написанных в середине прошлого века.
        !           967: Назовите АЛЬФУ двумя словами.
        !           968: 
        !           969: Ответ:
        !           970: Капитанская дочка.
        !           971: 
        !           972: Комментарий:
        !           973: АЛЬФА - капитанская дочка. Пеппи была дочкой капитана Эфраима
        !           974: Длинныйчулок.
        !           975: 
        !           976: Источник:
        !           977:    1. Н.А. Каланов. Словарь морского жаргона. - М.: Моркнига, 2010.
        !           978:    2. http://ru.wikipedia.org/wiki/Пеппи_Длинныйчулок
        !           979: 
        !           980: Автор:
        !           981: Артем Матухно (Одесса)
        !           982: 
        !           983: Вопрос 10:
        !           984: В одном кроссворде слово "рикша" определяется как "таксист, которого
        !           985: [ДВА СЛОВА ПРОПУЩЕНО]". А кого [ДВА СЛОВА ПРОПУЩЕНО], согласно русской
        !           986: пословице?
        !           987: 
        !           988: Ответ:
        !           989: Волка.
        !           990: 
        !           991: Комментарий:
        !           992: Пропущены слова "ноги кормят". Пословица - "Волка ноги кормят".
        !           993: 
        !           994: Источник:
        !           995:    1. http://www.kotvet.ru/odn/145
        !           996:    2. http://slovarick.ru/205/
        !           997: 
        !           998: Автор:
        !           999: Артем Матухно (Одесса)
        !          1000: 
        !          1001: Вопрос 11:
        !          1002: По одной из версий, ИКС - это переиначенное на английский лад выражение,
        !          1003: с помощью которого французские портовые грузчики просили коллег о
        !          1004: помощи. Какое слово мы заменили словом "ИКС"?
        !          1005: 
        !          1006: Ответ:
        !          1007: Mayday.
        !          1008: 
        !          1009: Зачет:
        !          1010: Мэйдэй; мэдэ.
        !          1011: 
        !          1012: Комментарий:
        !          1013: Речь идет о сигнале помощи "Mayday" [мэйдэй], который представляет собой
        !          1014: искаженное французское "m'aide" [мэд] - "помоги мне".
        !          1015: 
        !          1016: Источник:
        !          1017: Ю.И. Рылёв. 6000 изобретений XX и XXI веков, изменившие мир.
        !          1018: http://www.flibusta.is/b/377039/read
        !          1019: 
        !          1020: Автор:
        !          1021: Артем Матухно (Одесса)
        !          1022: 
        !          1023: Вопрос 12:
        !          1024: Рассказывают, что перед съемками "Терминатора" Арнольд Шварценеггер
        !          1025: специально подолгу проводил время на стрельбище, дабы научиться спускать
        !          1026: курок, не ДЕЛАЯ ЭТО. Как называется игра, в которой нельзя ДЕЛАТЬ ЭТО?
        !          1027: 
        !          1028: Ответ:
        !          1029: Гляделки.
        !          1030: 
        !          1031: Комментарий:
        !          1032: ДЕЛАТЬ ЭТО - моргать. Шварценеггер полагал, что роботы не моргают.
        !          1033: 
        !          1034: Источник:
        !          1035:    1. https://tjournal.ru/p/terminator-30-years
        !          1036:    2. http://ru.wiktionary.org/wiki/гляделки
        !          1037: 
        !          1038: Автор:
        !          1039: Артем Матухно (Одесса)
        !          1040: 
        !          1041: Тур:
        !          1042: Второй игровой день. 2 тур
        !          1043: 
        !          1044: Дата:
        !          1045: 11-Nov-2016
        !          1046: 
        !          1047: Редактор:
        !          1048: Дмитрий Петров и Михаил Локшин (Санкт-Петербург)
        !          1049: 
        !          1050: Инфо:
        !          1051: Редакторы благодарят за тестирование вопросов и ценные замечания:
        !          1052: Алексея и Марию Трефиловых, Артема Корсуна, Владимира Салия, Кристину
        !          1053: Кораблину, Александра Мудрого, Сергея Терентьева, Сергея Лобачёва,
        !          1054: Ярослава Косырева, команду Павла Ершова и лично Павла Ершова, а также
        !          1055: команды "Кадис.ру", "Ноев ковчег" и "Постмодернистское название".
        !          1056: 
        !          1057: Вопрос 1:
        !          1058: В Крыму первый ОН появился благодаря Максимилиану Волошину. По мнению
        !          1059: Сергея Федина, по уму встречают только на НЕМ. Назовите ЕГО двумя
        !          1060: словами.
        !          1061: 
        !          1062: Ответ:
        !          1063: Нудистский пляж.
        !          1064: 
        !          1065: Комментарий:
        !          1066: Там по одежке не встречают. Волошину принадлежат труды "Блики. Нагота" и
        !          1067: "Блики. Маски. Нагота". И даже Ленин считал, что в движении нудистов
        !          1068: есть "здоровое пролетарское начало".
        !          1069: 
        !          1070: Источник:
        !          1071:    1. http://ru.wikipedia.org/wiki/Натуризм
        !          1072:    2. http://www.gramma.ru/RST/?id=3.810
        !          1073: 
        !          1074: Автор:
        !          1075: Михаил Локшин (Санкт-Петербург)
        !          1076: 
        !          1077: Вопрос 2:
        !          1078: Сахарную голову можно поджечь, если с ней СДЕЛАТЬ ЭТО. Впрочем, порой
        !          1079: ЭТО ДЕЛАЮТ не только с сахарной. Какие два слова на одну и ту же букву
        !          1080: мы заменили словами "СДЕЛАТЬ ЭТО"?
        !          1081: 
        !          1082: Ответ:
        !          1083: Посыпать пеплом.
        !          1084: 
        !          1085: Комментарий:
        !          1086: Пепел (в частности, табачный) содержит соли калия, в том числе поташ,
        !          1087: катализирующие горение сахарозы.
        !          1088: 
        !          1089: Источник:
        !          1090:    1. http://www.youtube.com/watch?v=bckN7iMhjmg
        !          1091:    2. http://p-i-f.livejournal.com/6982595.html
        !          1092: 
        !          1093: Автор:
        !          1094: Михаил Локшин (Санкт-Петербург)
        !          1095: 
        !          1096: Вопрос 3:
        !          1097: Виктор Гюго провел несколько лет в изгнании на скалистом острове Гернси.
        !          1098: Бодлер насмешливо писал, что терзаемый лишь уколами тщеславия Гюго любил
        !          1099: воображать себя... Кем?
        !          1100: 
        !          1101: Ответ:
        !          1102: Прометеем.
        !          1103: 
        !          1104: Комментарий:
        !          1105: Виктор Гюго, не принявший власть Наполеона III, был своего рода
        !          1106: бунтарем, как и Прометей. А Бодлер был весьма циничен и товарищей по
        !          1107: перу недолюбливал.
        !          1108: 
        !          1109: Источник:
        !          1110: А. Труайя. Бодлер. http://www.flibusta.is/b/291671/read
        !          1111: 
        !          1112: Автор:
        !          1113: Дмитрий Петров (Санкт-Петербург)
        !          1114: 
        !          1115: Вопрос 4:
        !          1116: Живший в XVI веке римский папа Сикст V любил грандиозные проекты. Так,
        !          1117: папа обещал оплатить ЕЕ создание, если ОНА доберется до цели. Для НЕЕ
        !          1118: была разработана специальная система сигналов. Назовите ЕЕ.
        !          1119: 
        !          1120: Ответ:
        !          1121: Непобедимая армада.
        !          1122: 
        !          1123: Зачет:
        !          1124: Испанская армада; Великая армада.
        !          1125: 
        !          1126: Комментарий:
        !          1127: Сикст V обещал оплатить создание армады после высадки испанцев в Англии.
        !          1128: В состав флота входили корабли нескольких государств. Специально для
        !          1129: Армады был создан интернациональный свод морских сигналов.
        !          1130: 
        !          1131: Источник:
        !          1132:    1. Р. Хьюз. Рим. История города: его культура, облик, люди. - М.:
        !          1133: АСТ: CORPUS, 2014. - С. 288.
        !          1134:    2. http://ru.wikipedia.org/wiki/Непобедимая_армада
        !          1135: 
        !          1136: Автор:
        !          1137: Дмитрий Петров (Санкт-Петербург)
        !          1138: 
        !          1139: Вопрос 5:
        !          1140: Внимание, в вопросе есть замена.
        !          1141:    Премьер-министр Турции на полном серьезе заявил: взгляды турецких
        !          1142: мусульман отличаются от взглядов боевиков ИГИЛ на целых ДВА ИКС.
        !          1143: Ответьте точно, что мы заменили словами "ДВА ИКС".
        !          1144: 
        !          1145: Ответ:
        !          1146: 360 градусов.
        !          1147: 
        !          1148: Комментарий:
        !          1149: Ахмет Давутоглу сказал, что взгляды турецких мусульман отличаются от
        !          1150: взглядов террористов не на 180, а на целых 360 градусов. На всякий
        !          1151: случай упоминаем, что террористическая организация "Исламское
        !          1152: Государство" запрещена на территории РФ.
        !          1153: 
        !          1154: Источник:
        !          1155: http://www.middleeasteye.net/news/turkish-pm-mocked-over-360-degrees-difference-slip-190994676
        !          1156: 
        !          1157: Автор:
        !          1158: Дмитрий Петров (Санкт-Петербург)
        !          1159: 
        !          1160: Вопрос 6:
        !          1161: Пикап Toyota Hilux [тойОта хАйлакс] надежен и прост в ремонте. По словам
        !          1162: одного американского военнослужащего, Toyota Hilux стал автомобильным
        !          1163: эквивалентом ЕГО. Назовите ЕГО двумя словами.
        !          1164: 
        !          1165: Ответ:
        !          1166: Автомат Калашникова.
        !          1167: 
        !          1168: Комментарий:
        !          1169: Toyota Hilux из-за технических характеристик чаще всего используется как
        !          1170: импровизированная боевая машина в локальных конфликтах.
        !          1171: 
        !          1172: Источник:
        !          1173:    1. http://www.businessinsider.com/why-isis-uses-toyota-trucks-2015-10
        !          1174:    2. http://ru.wikipedia.org/wiki/Toyota_Hilux
        !          1175: 
        !          1176: Автор:
        !          1177: Дмитрий Петров (Санкт-Петербург)
        !          1178: 
        !          1179: Вопрос 7:
        !          1180: На фабрике ЕГО отца делали детали к самолетам "Мицубиси". Герой ЕГО
        !          1181: произведения говорит: "Самолет - это прекрасная мечта, а конструктор -
        !          1182: тот, кто ее воплощает". Назовите ЕГО.
        !          1183: 
        !          1184: Ответ:
        !          1185: [Хаяо] Миядзаки.
        !          1186: 
        !          1187: Комментарий:
        !          1188: Разнообразные летательные аппараты, в первую очередь самолеты, можно
        !          1189: увидеть во множестве его произведений: "Небесный замок Лапута", "Ветер
        !          1190: крепчает", "Навсикая из Долины Ветров", "Порко Россо" и других. Фразу из
        !          1191: вопроса произносит авиаконструктор из фильма "Ветер крепчает".
        !          1192: 
        !          1193: Источник:
        !          1194:    1. http://otium.su/xayao-miadzaki-velikij-fantazyor/
        !          1195:    2. http://ru.wikipedia.org/wiki/Миядзаки,_Хаяо
        !          1196:    3. Мультфильм "Ветер крепчает" (2013), реж. Хаяо Миядзаки, 12-я
        !          1197: минута. http://ru.wiki2.org/wiki/Ветер_крепчает?s=The%20Wind%20Rises
        !          1198: 
        !          1199: Автор:
        !          1200: Михаил Локшин (Санкт-Петербург)
        !          1201: 
        !          1202: Вопрос 8:
        !          1203: Разработанный Майком Томпсоном прибор для поиска сигнала Wi-Fi выглядит
        !          1204: как ОНА. Считается, что название деревни Аргури близ горы Арарат связано
        !          1205: с НЕЙ. Назовите ЕЕ одним словом.
        !          1206: 
        !          1207: Ответ:
        !          1208: Лоза.
        !          1209: 
        !          1210: Комментарий:
        !          1211: Дизайнер решил, что поиски сигнала Wi-Fi подобны поискам воды
        !          1212: лозоходцами. Как известно, на горе Арарат зреет сладкий виноград.
        !          1213: 
        !          1214: Источник:
        !          1215:    1. http://www.etoday.ru/2011/10/ustroystvo-dlya-poiska-wifi--.php
        !          1216:    2. http://ru.wikipedia.org/wiki/Лозоходство
        !          1217:    3. http://ru.wikipedia.org/wiki/Арарат
        !          1218:    4. http://ru.wikisource.org/wiki/ЭСБЕ/Аргури
        !          1219: 
        !          1220: Автор:
        !          1221: Дмитрий Петров (Санкт-Петербург)
        !          1222: 
        !          1223: Вопрос 9:
        !          1224: Механизмы работы иммунитета изучены недостаточно. Ученый Александр
        !          1225: ХаджИдис говорит, что иммунитет - это тонкая и пока еще ТАКАЯ ОНА.
        !          1226: Назовите ТАКУЮ ЕЕ.
        !          1227: 
        !          1228: Ответ:
        !          1229: Темная материя.
        !          1230: 
        !          1231: Комментарий:
        !          1232: Темная материя - гипотетическая форма материи, которая не испускает
        !          1233: электромагнитного излучения и напрямую не взаимодействует с ним. По
        !          1234: словам ученого, "иммунитет - это тонкая и пока еще темная материя".
        !          1235: 
        !          1236: Источник:
        !          1237:    1. http://doctorpiter.ru/articles/11136/
        !          1238:    2. http://ru.wikipedia.org/wiki/Тёмная_материя
        !          1239: 
        !          1240: Автор:
        !          1241: Михаил Локшин (Санкт-Петербург)
        !          1242: 
        !          1243: Вопрос 10:
        !          1244: Как-то раз сотрудники одного английского музея пришли на работу и
        !          1245: обнаружили, что ОНА XIX века до сих пор действует. Назовите ЕЕ.
        !          1246: 
        !          1247: Ответ:
        !          1248: Мышеловка.
        !          1249: 
        !          1250: Комментарий:
        !          1251: Несмотря на отсутствие приманки, музейная мышь забралась в старинную
        !          1252: мышеловку и не смогла оттуда выбраться.
        !          1253: 
        !          1254: Источник:
        !          1255: http://blogs.reading.ac.uk/merl/2016/02/03/155-year-old-mouse-trap-claims-its-latest-victim/
        !          1256: 
        !          1257: Автор:
        !          1258: Дмитрий Петров (Санкт-Петербург)
        !          1259: 
        !          1260: Вопрос 11:
        !          1261: Единственное слово, заимствованное английским языком из швейцарского
        !          1262: ретороманского, обозначает ЕЕ. Один из методов предсказания ИХ тоже
        !          1263: происходит из Швейцарии. Назовите ЕЕ.
        !          1264: 
        !          1265: Ответ:
        !          1266: Лавина.
        !          1267: 
        !          1268: Комментарий:
        !          1269: Английское слово "avalanche" [Эвэланш] - "лавина" - пришло из
        !          1270: швейцарского ретороманского через французский. Один из методов оценки
        !          1271: устойчивости снежно-ледового покрова был разработан в швейцарской армии.
        !          1272: 
        !          1273: Источник:
        !          1274:    1. Г. Доррен. Лингво. Языковой пейзаж Европы.
        !          1275: http://www.flibusta.is/b/446640/read
        !          1276:    2. http://ru.wikipedia.org/wiki/Лавина
        !          1277: 
        !          1278: Автор:
        !          1279: Дмитрий Петров (Санкт-Петербург)
        !          1280: 
        !          1281: Вопрос 12:
        !          1282: Древние китайцы верили, что ракУшки каУри приносят удачу, и придали
        !          1283: ЭТОМУ форму ракУшки каУри. Если вы решите приготовить ЭТО, лучше
        !          1284: использовать лазерный принтер - иначе будет трудно что-либо разобрать.
        !          1285: Назовите ЭТО тремя словами.
        !          1286: 
        !          1287: Ответ:
        !          1288: Печенье с предсказанием.
        !          1289: 
        !          1290: Зачет:
        !          1291: Печенье с предсказаниями.
        !          1292: 
        !          1293: Комментарий:
        !          1294: Выпекая из муки съедобное подобие каури, китайцы верили, что это
        !          1295: принесет им удачу. Если использовать струйный принтер, шариковую или
        !          1296: гелевую ручку, при высокой температуре текст на бумаге может смазаться.
        !          1297: 
        !          1298: Источник:
        !          1299: http://www.orientalica.com/kitayskoe-pechene-s-predskazaniyami-retseptyi-i-istoriya
        !          1300: 
        !          1301: Автор:
        !          1302: Михаил Локшин (Санкт-Петербург)
        !          1303: 
        !          1304: Тур:
        !          1305: Второй игровой день. 3 тур
        !          1306: 
        !          1307: Дата:
        !          1308: 11-Nov-2016
        !          1309: 
        !          1310: Редактор:
        !          1311: Александр Мудрый (Черновцы)
        !          1312: 
        !          1313: Инфо:
        !          1314: Редактор благодарит за тестирование и ценные замечания: Ростислава
        !          1315: Гимчинского, Владимира Городецкого, Александра Зинченко, Ирину Зубкову,
        !          1316: Андрея Кокуленко, Николая Константинова, Артема Корсуна, Александра
        !          1317: Кудрявцева, Константина Науменко, Алексея и Марию Трефиловых, Игоря
        !          1318: Тюнькина, Ульяну Фабричнину, а также команду "Прометей" (Черновцы).
        !          1319: 
        !          1320: Вопрос 1:
        !          1321: Более четверти века в этом городе не функционировали полтора десятка
        !          1322: станций метрополитена. Назовите этот город.
        !          1323: 
        !          1324: Ответ:
        !          1325: Восточный Берлин.
        !          1326: 
        !          1327: Зачет:
        !          1328: Берлин.
        !          1329: 
        !          1330: Комментарий:
        !          1331: После сооружения Берлинской стены были закрыты семь станций на линии U6
        !          1332: и восемь станций на линии U8 в связи с тем, что эти линии шли из
        !          1333: западного сектора в западный через восточную часть. Было принято решение
        !          1334: не разрывать линии западного метрополитена, а только закрыть станции,
        !          1335: находящиеся в восточном секторе. Поезда через эти станции двигались, не
        !          1336: останавливаясь.
        !          1337: 
        !          1338: Источник:
        !          1339:    1. http://ru.wikipedia.org/wiki/Берлинский_метрополитен
        !          1340:    2. http://forum.tr.ru/read.php?5,174129,page=all
        !          1341: 
        !          1342: Автор:
        !          1343: Александр Мудрый (Черновцы)
        !          1344: 
        !          1345: Вопрос 2:
        !          1346: На мемориальной доске Траяну Поповичу, который был мэром Черновцов в
        !          1347: середине прошлого века, есть число 19600. Сергей Воронцов назвал
        !          1348: Поповича "буковинским ИМ". Назовите ЕГО.
        !          1349: 
        !          1350: Ответ:
        !          1351: [Оскар] Шиндлер.
        !          1352: 
        !          1353: Комментарий:
        !          1354: В 1941 году Попович отказался выполнить приказ о депортации черновицких
        !          1355: евреев в лагеря Транснистрии. Мало того, он убедил вышестоящие власти в
        !          1356: своей правоте и добился изменения решения: 19600 черновицких евреев
        !          1357: остались в городе.
        !          1358: 
        !          1359: Источник:
        !          1360:    1. http://gazeta.zn.ua/SOCIETY/bukovinskiy_shindler.html
        !          1361:    2. https://pogliad.ua/news/chernivtsi/u-chernivcyah-vidkrili-memorialnu-doshku-trayanu-popovichu-280684
        !          1362: 
        !          1363: Автор:
        !          1364: Александр Мудрый (Черновцы)
        !          1365: 
        !          1366: Вопрос 3:
        !          1367: Большинство украинских телеканалов транслируются со спутников "Amos"
        !          1368: [Эймос], "Sirius" [сИриус] и "HotBird" [хотбёд]. Поэтому самый
        !          1369: распространенный на Украине тип спутниковых антенн получил сказочное
        !          1370: прозвище "он". В одном из слов предыдущего предложения мы пропустили
        !          1371: пять букв. Напишите это слово в исходном виде.
        !          1372: 
        !          1373: Ответ:
        !          1374: Горыныч.
        !          1375: 
        !          1376: Комментарий:
        !          1377: Чтобы поймать все три спутника, на тарелку устанавливают три конвертера,
        !          1378: которые в просторечии называют головками. Такая "трехголовая" антенна и
        !          1379: получила название в честь Змея из русских сказок.
        !          1380: 
        !          1381: Источник:
        !          1382:    1. http://www.satsis.info/forum (поиск по слову "горыныч")
        !          1383:    2. http://www.satsputnik.ru/vybrat-oborudovanie-dlya-prosmotra-besplatnogo-sputnikovogo-televideniya/
        !          1384: 
        !          1385: Автор:
        !          1386: Александр Мудрый (Черновцы)
        !          1387: 
        !          1388: Вопрос 4:
        !          1389: В первом тайме матча Евро-2016 Италия - Германия обе команды практически
        !          1390: не предпринимали атакующих действий. Денис Казанский назвал этот тайм
        !          1391: "ПРОПУСК ГарОнне". Заполните пропуск двумя или тремя словами.
        !          1392: 
        !          1393: Ответ:
        !          1394: Стояние на реке.
        !          1395: 
        !          1396: Зачет:
        !          1397: Стоянием на реке; стояние на; стоянием на.
        !          1398: 
        !          1399: Комментарий:
        !          1400: Матч проходил в Бордо, расположенном на реке Гаронне, и комментатор
        !          1401: сравнил такой футбол с известными событиями российской истории.
        !          1402: 
        !          1403: Источник:
        !          1404: Трансляция четвертьфинала Евро-2016 Италия - Германия на канале "Матч!
        !          1405: Футбол 1", эфир от 02.07.2016 г.
        !          1406: 
        !          1407: Автор:
        !          1408: Александр Мудрый (Черновцы)
        !          1409: 
        !          1410: Вопрос 5:
        !          1411: Одна из версий возникновения ЭТОГО - сбой в одном из отделов мозга. В
        !          1412: такой момент новая информация поступает в подкорковые ядра, отвечающие
        !          1413: за ее переработку, не напрямую, а через другую зону. Назовите ЭТО.
        !          1414: 
        !          1415: Ответ:
        !          1416: Дежавю.
        !          1417: 
        !          1418: Комментарий:
        !          1419: Из-за сбоя в парагиппокампальной извилине новая информация поступает в
        !          1420: подкорковые ядра через правую височную долю, отвечающую за средне- и
        !          1421: долговременную память. Поэтому человек воспринимает эту новую информацию
        !          1422: как уже виденную.
        !          1423: 
        !          1424: Источник:
        !          1425: http://pikabu.ru/story/prostoe_i_ponyatnoe_obyasnenie_pochemu_myi_ispyityivaem_dezhavyu_4484414
        !          1426: 
        !          1427: Автор:
        !          1428: Ростислав Гимчинский (Черновцы)
        !          1429: 
        !          1430: Вопрос 6:
        !          1431: После того как в болотах графства Кембриджшир было обнаружено деревянное
        !          1432: колесо, эту территорию прозвали местными ИМИ. Назовите ИХ одним словом.
        !          1433: 
        !          1434: Ответ:
        !          1435: Помпеи.
        !          1436: 
        !          1437: Комментарий:
        !          1438: В болотах при отсутствии доступа кислорода очень хорошо сохраняются
        !          1439: археологические находки. Место раскопок поселения близ Питерборо, в
        !          1440: котором нашли дубовое колесо возрастом 3000 лет, сами археологи назвали
        !          1441: самым богатым памятником бронзового века в Великобритании и сравнили с
        !          1442: легендарными Помпеями.
        !          1443: 
        !          1444: Источник:
        !          1445: https://www.theguardian.com/science/2016/feb/19/archaeologists-excavate-bronze-age-wheel-cambridgeshire
        !          1446: 
        !          1447: Автор:
        !          1448: Александр Мудрый (Черновцы)
        !          1449: 
        !          1450: Вопрос 7:
        !          1451: В одном сериале маленькая компания, разрабатывающая инновационную
        !          1452: технологию, узнаёт, что у нее появился серьезный конкурент. Один из
        !          1453: персонажей с грустью говорит, что их шансы опередить этого конкурента
        !          1454: ЧРЕЗВЫЧАЙНО МАЛЫ. Какое соотношение мы заменили словами "ЧРЕЗВЫЧАЙНО
        !          1455: МАЛЫ"?
        !          1456: 
        !          1457: Ответ:
        !          1458: Один к гуглу.
        !          1459: 
        !          1460: Зачет:
        !          1461: 1:10^100.
        !          1462: 
        !          1463: Комментарий:
        !          1464: Конкурентом этой компании был "Google". Персонаж скаламбурил, понимая,
        !          1465: что шансы небольшого стартапа обогнать IT-гиганта очень невелики.
        !          1466: 
        !          1467: Источник:
        !          1468: Телесериал "Два с половиной человека", s11e16.
        !          1469: 
        !          1470: Автор:
        !          1471: Александр Мудрый (Черновцы)
        !          1472: 
        !          1473: Вопрос 8:
        !          1474: Внимание, ИКС в вопросе - замена.
        !          1475:    Получивший образование в Париже Тадеуш Костюшко устроился ИКСОМ,
        !          1476: чтобы быть поближе к любимой девушке. А заглавный персонаж известного
        !          1477: произведения стал ИКСОМ, чтобы быть поближе к своему врагу. Назовите
        !          1478: фамилию этого персонажа.
        !          1479: 
        !          1480: Ответ:
        !          1481: Дубровский.
        !          1482: 
        !          1483: Комментарий:
        !          1484: ИКС - гувернёр, учитель французского. Костюшко устроился гувернёром в
        !          1485: дом богатого помещика Сосновского, в дочь которого Людвику был влюблен.
        !          1486: 
        !          1487: Источник:
        !          1488:    1. Документальный фильм "Тадеуш Костюшко - польский генерал".
        !          1489:    2. http://ru.wikipedia.org/wiki/Дубровский_(роман)
        !          1490: 
        !          1491: Автор:
        !          1492: Александр Мудрый (Черновцы)
        !          1493: 
        !          1494: Вопрос 9:
        !          1495: Скоростной хоккеист Виталий Абрамов в прошлом сезоне ярко ворвался в
        !          1496: одну из юниорских лиг. Павел Климовицкий назвал Абрамова ИМ. Сам Абрамов
        !          1497: видел ЕГО, будучи в школе. Назовите ЕГО двумя словами.
        !          1498: 
        !          1499: Ответ:
        !          1500: Челябинский метеор.
        !          1501: 
        !          1502: Зачет:
        !          1503: Челябинский метеорит.
        !          1504: 
        !          1505: Комментарий:
        !          1506: Абрамов был признан лучшим новичком Главной юниорской хоккейной лиги
        !          1507: Квебека (QMJHL). Поскольку Виталий из Челябинска, то его не преминули
        !          1508: сравнить с метеоритом, пролетевшим над городом утром 15 февраля 2013
        !          1509: года. В это время 14-летний Абрамов, как и положено, находился в школе.
        !          1510: 
        !          1511: Источник:
        !          1512:    1. http://www.sport-express.ru/hockey/nhl/reviews/1014473/
        !          1513:    2. http://www.nhl.com/ice/ru/news.htm?id=886932
        !          1514: 
        !          1515: Автор:
        !          1516: Александр Мудрый (Черновцы)
        !          1517: 
        !          1518: Вопрос 10:
        !          1519: Наблюдение за одной экзопланетой поставило под сомнение эффективность
        !          1520: процесса циркуляризации. Заметка об этом называлась "ЧУДАКОВАТАЯ
        !          1521: экзопланета задает вопросы". Какое слово мы заменили словом
        !          1522: "ЧУДАКОВАТАЯ"?
        !          1523: 
        !          1524: Ответ:
        !          1525: Эксцентричная.
        !          1526: 
        !          1527: Зачет:
        !          1528: Эксцентрическая.
        !          1529: 
        !          1530: Комментарий:
        !          1531: Эта планета расположена в созвездии Большой Медведицы и представляет
        !          1532: собой газовый гигант размером с Юпитер. Она вращается вокруг своей
        !          1533: звезды по очень вытянутой, т.е. эксцентричной орбите. Процесс
        !          1534: циркуляризации состоит в "выравнивании" орбиты планеты, постепенном
        !          1535: приближении ее к круговой. Но в данном случае он происходит гораздо
        !          1536: медленнее, чем предсказывает теория.
        !          1537: 
        !          1538: Источник:
        !          1539:    1. http://www.nkj.ru/news/28463/
        !          1540:    2. http://dic.academic.ru/dic.nsf/dic_synonims/201591/
        !          1541: 
        !          1542: Автор:
        !          1543: Александр Мудрый (Черновцы)
        !          1544: 
        !          1545: Вопрос 11:
        !          1546: Рассказывая о заядлом рыбаке, Вашингтон Ирвинг пишет, что тот живет
        !          1547: жизнью настоящего ЕГО. ОНИ, вероятно, происходят от кистепёрых рыб.
        !          1548: Назовите ИХ двухкоренным словом.
        !          1549: 
        !          1550: Ответ:
        !          1551: Земноводные.
        !          1552: 
        !          1553: Комментарий:
        !          1554: Рыбак из рассказа Ирвинга в воде и около нее проводил не меньше времени,
        !          1555: чем на суше. Биологи полагают, что кистепёрые рыбы дали начало
        !          1556: земноводным и первыми из позвоночных вышли на сушу.
        !          1557: 
        !          1558: Источник:
        !          1559:    1. В. Ирвинг. Происшествие с черным рыбаком.
        !          1560: http://www.flibusta.is/b/70482/read
        !          1561:    2. http://ru.wikipedia.org/wiki/Происхождение_земноводных
        !          1562:    3. http://ru.wikipedia.org/wiki/Кистепёрые_рыбы
        !          1563: 
        !          1564: Автор:
        !          1565: Александр Мудрый (Черновцы)
        !          1566: 
        !          1567: Вопрос 12:
        !          1568: Назовите роман, первый перевод которого на русский язык выполнила в 1936
        !          1569: году Евгения Калашникова.
        !          1570: 
        !          1571: Ответ:
        !          1572: "Прощай, оружие!".
        !          1573: 
        !          1574: Комментарий:
        !          1575: Вот такое любопытное совпадение. Кстати, по мнению Корнея Чуковского,
        !          1576: этот перевод принадлежит к высшим достижениям советского переводческого
        !          1577: искусства.
        !          1578: 
        !          1579: Источник:
        !          1580: http://ru.wikipedia.org/wiki/Калашникова,_Евгения_Давыдовна
        !          1581: 
        !          1582: Автор:
        !          1583: Александр Мудрый (Черновцы)
        !          1584: 
        !          1585: Тур:
        !          1586: Третий игровой день. 1 тур
        !          1587: 
        !          1588: Дата:
        !          1589: 09-Dec-2016
        !          1590: 
        !          1591: Редактор:
        !          1592: Антон Волосатов и Константин Сахаров (Ивантеевка)
        !          1593: 
        !          1594: Инфо:
        !          1595: Помощи в подготовке уделили свое ценное время: Никита Коровин (Манила),
        !          1596: Дмитрий Дягилев, Татьяна Левченко, Галина Пактовская, Серафим Шибанов
        !          1597: (все - Москва), Александр Усков и Олег Михеев (оба - Краснодар), Николай
        !          1598: Коврижных (Киров), Екатерина Сахарова (Ивантеевка).
        !          1599: 
        !          1600: Вопрос 1:
        !          1601: [Ведущему: отточие не озвучивать.]
        !          1602:    Заметка о чемпионате по какой игре вышла под заголовком "... Без
        !          1603: покемонов"?
        !          1604: 
        !          1605: Ответ:
        !          1606: Го.
        !          1607: 
        !          1608: Комментарий:
        !          1609: Мобильная игра "Pokémon Go" крайне популярна (или была таковой
        !          1610: несколько месяцев назад), но и поклонников интеллектуальной игры го в
        !          1611: мире насчитывают до 60 миллионов. Больше вопросов о покемонах и по
        !          1612: заголовкам в туре не будет!
        !          1613: 
        !          1614: Источник:
        !          1615: https://www.championat.com/other/article-252640-v-sankt-peterburge-prohodit-60-j-evropejskij-kongress-go.html
        !          1616: 
        !          1617: Автор:
        !          1618: Константин Сахаров (Ивантеевка)
        !          1619: 
        !          1620: Вопрос 2:
        !          1621: Однажды автор вопроса с друзьями играли в "Угадай кто", и одному из
        !          1622: игроков достался популярный персонаж американского мультсериала.
        !          1623: Воспроизведите первый вопрос, который задал о себе игрок.
        !          1624: 
        !          1625: Ответ:
        !          1626: "Я в данный момент умер?".
        !          1627: 
        !          1628: Зачет:
        !          1629: По упоминанию смерти или жизни.
        !          1630: 
        !          1631: Комментарий:
        !          1632: Кенни из "South Park" известен тем, что постоянно умирает и воскресает,
        !          1633: поэтому вопрос поставил партнеров в тупик.
        !          1634: 
        !          1635: Источник:
        !          1636: ЛОАВ.
        !          1637: 
        !          1638: Автор:
        !          1639: Константин Сахаров (Ивантеевка)
        !          1640: 
        !          1641: Вопрос 3:
        !          1642: Американская чемпионка мира в детстве нередко терпела оскорбления на
        !          1643: религиозной почве. Поэтому вид спорта был выбран такой, чтобы не
        !          1644: испытывать проблем с экипировкой. Назовите этот вид спорта.
        !          1645: 
        !          1646: Ответ:
        !          1647: Фехтование.
        !          1648: 
        !          1649: Комментарий:
        !          1650: Одежда девушек-мусульман не должна открывать руки и ноги. В фехтовании
        !          1651: тело и даже лицо спортсмена полностью закрыто униформой. Так что мама
        !          1652: разрешила ИбтихАдж МухаммАд заниматься спортом, увидев подходящие
        !          1653: костюмы фехтовальщиц. В Рио-де-Жанейро Мухаммад стала первой
        !          1654: американкой, которая выступала на Олимпиаде в хиджабе.
        !          1655: 
        !          1656: Источник:
        !          1657:    1. http://www.sports.ru/tribuna/blogs/innuendo/1020658.html
        !          1658:    2. http://ru.wikipedia.org/wiki/Мухаммад,_Ибтихадж
        !          1659: 
        !          1660: Автор:
        !          1661: Константин Сахаров (Ивантеевка)
        !          1662: 
        !          1663: Вопрос 4:
        !          1664: Белый крест на зеленом фоне - таким предлагали сделать флаг ЕЕ. Назовите
        !          1665: ЕЕ.
        !          1666: 
        !          1667: Ответ:
        !          1668: Гренландия.
        !          1669: 
        !          1670: Комментарий:
        !          1671: Гренландия - зависимая территория Дании, и собственный флаг ей
        !          1672: предлагали сделать по аналогии со всеми скандинавскими, но с учетом
        !          1673: "зеленого" названия территории. Вероятно, из-за стремления подчеркнуть
        !          1674: самостоятельность нынешний флаг скандинавского креста не содержит.
        !          1675: 
        !          1676: Источник:
        !          1677: http://ru.wikipedia.org/wiki/Флаг_Гренландии
        !          1678: 
        !          1679: Автор:
        !          1680: Константин Сахаров (Ивантеевка)
        !          1681: 
        !          1682: Вопрос 5:
        !          1683: Обсуждая фильм 2015 года, научный сотрудник Артем Климчук призвал ценить
        !          1684: знания, потому что они могут пригодиться в самый неожиданный момент. По
        !          1685: его мнению, от негативных коннотаций должно избавиться слово, означающее
        !          1686: специалиста. Напишите это слово.
        !          1687: 
        !          1688: Ответ:
        !          1689: Ботаник.
        !          1690: 
        !          1691: Комментарий:
        !          1692: В фильме "Марсианин" главный герой - по основной профессии ботаник -
        !          1693: сумел применить свои знания и выжить долгое время на Марсе. "Если
        !          1694: понятие "ботаник" воспринимать не как человека, который занимается
        !          1695: растениями, а как человека, который много чего знает и хорошо учился в
        !          1696: институте, то краткая мораль фильма - будьте ботаниками. Это вас
        !          1697: спасет".
        !          1698: 
        !          1699: Источник:
        !          1700: http://www.the-village.ru/village/weekend/oba/223751-the-martian-mfti
        !          1701: 
        !          1702: Автор:
        !          1703: Константин Сахаров (Ивантеевка)
        !          1704: 
        !          1705: Вопрос 6:
        !          1706: Комментируя итоги выборов осенью 2016 года, Эльвира Набиуллина заявила:
        !          1707: "Мы, конечно, все помним про песню". Из какого города группа,
        !          1708: исполнившая эту песню?
        !          1709: 
        !          1710: Ответ:
        !          1711: Из Владивостока.
        !          1712: 
        !          1713: Комментарий:
        !          1714: Выбор городов для размещения на новых рублевых купюрах завершился в
        !          1715: пользу Севастополя и Дальнего Востока. Решение о том, какой из городов
        !          1716: окажется на какой купюре, будет принимать совет директоров Центробанка
        !          1717: России (Набиуллина - его глава). Песня, о которой идет речь, -
        !          1718: "Владивосток 2000" группы "Мумий Тролль".
        !          1719: 
        !          1720: Источник:
        !          1721: http://www.vesti.ru/doc.html?id=2809499
        !          1722: 
        !          1723: Автор:
        !          1724: Константин Сахаров (Ивантеевка)
        !          1725: 
        !          1726: Вопрос 7:
        !          1727: В Эфиопии и Эритрее практикуется ОНА, включающая в себя такие стадии:
        !          1728: обжарка на углях, перемалывание в деревянной ступке, варка в котле,
        !          1729: перелив в другой сосуд и охлаждение, просеивание через фильтр из
        !          1730: конского волоса и наконец, разлив в кипящем состоянии. Назовите ЕЕ двумя
        !          1731: словами.
        !          1732: 
        !          1733: Ответ:
        !          1734: Кофейная церемония.
        !          1735: 
        !          1736: Комментарий:
        !          1737: Мы попытались дать описание, напоминающее более известную чайную
        !          1738: церемонию. Упомянутые африканские регионы известны своим кофе.
        !          1739: 
        !          1740: Источник:
        !          1741: http://www.intensocoffee.ru/articles.php?article=3_kofeinaja_ceremonia
        !          1742: 
        !          1743: Автор:
        !          1744: Антон Волосатов (Ивантеевка)
        !          1745: 
        !          1746: Вопрос 8:
        !          1747: На логотипе фирмы "Деловой звук" изображен несуществующий вид ЕГО -
        !          1748: скрипичный ключ. Назовите ЕГО двумя словами.
        !          1749: 
        !          1750: Ответ:
        !          1751: Галстучный узел.
        !          1752: 
        !          1753: Зачет:
        !          1754: Узел галстука; способ завязывания.
        !          1755: 
        !          1756: Комментарий:
        !          1757: Галстук - пожалуй, главнейший элемент делового стиля.
        !          1758: 
        !          1759: Источник:
        !          1760: http://www.business-sound.ru/
        !          1761: 
        !          1762: Автор:
        !          1763: Антон Волосатов (Ивантеевка)
        !          1764: 
        !          1765: Вопрос 9:
        !          1766: "Самый великий человек в мире - великий вождь Ким Ир Сен". Как
        !          1767: утверждает журнал "Максим", за эту фразу любого северокорейца отправили
        !          1768: бы в лагерь, так как ОНО не на месте. Напишите ЕГО.
        !          1769: 
        !          1770: Ответ:
        !          1771: Ким Ир Сен.
        !          1772: 
        !          1773: Зачет:
        !          1774: Имя [вождя].
        !          1775: 
        !          1776: Комментарий:
        !          1777: В КНДР имя Ким Ир Сена всегда должно стоять в начале предложения. Мы
        !          1778: нарушили это правило уже дважды. :-(
        !          1779: 
        !          1780: Источник:
        !          1781: http://www.maximonline.ru/longreads/get-smart/_article/north-korea/
        !          1782: 
        !          1783: Автор:
        !          1784: Илья Иванов (Москва)
        !          1785: 
        !          1786: Вопрос 10:
        !          1787: Когда ОНИ появились в Италии в начале прошлого века, за характерный
        !          1788: элемент ИХ стали называть "fumetti" [фумЕтти], что означает "дымок".
        !          1789: Ответьте одним словом, что такое ОНИ.
        !          1790: 
        !          1791: Ответ:
        !          1792: Комиксы.
        !          1793: 
        !          1794: Комментарий:
        !          1795: Итальянцам казалось, что спичбаблы в американских комиксах похожи на
        !          1796: облачка дыма, так что стали именовать так и сами комиксы.
        !          1797: 
        !          1798: Источник:
        !          1799:    1. Д.Е. Комм. Формулы страха. Введение в историю и теорию фильма
        !          1800: ужасов. http://www.flibusta.is/b/353326/read
        !          1801:    2. http://en.wikipedia.org/wiki/Italian_comics
        !          1802: 
        !          1803: Автор:
        !          1804: Руслан Хаиткулов (Москва)
        !          1805: 
        !          1806: Вопрос 11:
        !          1807: Искусствовед Эрвин ПанОфский пришел к выводу, что крылатый мальчик -
        !          1808: Эрот, а его связанные руки указывают на "вынужденное целомудрие".
        !          1809: Назовите заглавную героиню описанной картины.
        !          1810: 
        !          1811: Ответ:
        !          1812: Даная.
        !          1813: 
        !          1814: Комментарий:
        !          1815: Как известно, Зевс вступил в отношения с Данаей не какими-то привычными
        !          1816: методами, а в виде золотого дождя.
        !          1817: 
        !          1818: Источник:
        !          1819: http://ru.wikipedia.org/wiki/Даная_(картина_Рембрандта)
        !          1820: 
        !          1821: Автор:
        !          1822: Константин Сахаров (Ивантеевка)
        !          1823: 
        !          1824: Вопрос 12:
        !          1825: С 2016 года для НЕЕ официально введено ограничение в 45 секунд. В марте
        !          1826: 1946 года Рэй МИлланд от НЕЕ отказался, ограничившись поклоном, что до
        !          1827: сих пор остается уникальным случаем. Назовите ЕЕ максимально точно.
        !          1828: 
        !          1829: Ответ:
        !          1830: Речь победителя на вручении премии "Оскар".
        !          1831: 
        !          1832: Зачет:
        !          1833: По словам "речь" и "Оскар".
        !          1834: 
        !          1835: Комментарий:
        !          1836: Чтобы слишком не затягивать церемонию и избежать инцидентов, выступления
        !          1837: победителей регламентировали. Вручение наград традиционно проводится на
        !          1838: границе зимы и весны. Несмотря на то что в карьере Милланда это была
        !          1839: первая и последняя статуэтка, он проявил необычайную скромность. А
        !          1840: награжден он был за лучшую мужскую роль в фильме "Потерянный уик-энд".
        !          1841:    z-checkdb: Ограничение в 45 секунд действует с 2010 года, см.
        !          1842: http://www.hollywoodreporter.com/lists/oscars-watch-10-longest-acceptance-867667/item/greer-garson-10-longest-acceptance-867635
        !          1843: (Евгений Рубашкин).
        !          1844: 
        !          1845: Источник:
        !          1846:    1. http://oscar2016.ru/news/45-seconds/
        !          1847:    2. http://ru.wikipedia.org/wiki/Рэй_Милланд
        !          1848: 
        !          1849: Автор:
        !          1850: Антон Волосатов (Ивантеевка)
        !          1851: 
        !          1852: Тур:
        !          1853: Третий игровой день. 2 тур
        !          1854: 
        !          1855: Дата:
        !          1856: 09-Dec-2016
        !          1857: 
        !          1858: Редактор:
        !          1859: Александр Голиков и Людмила Полякова (Харьков)
        !          1860: 
        !          1861: Вопрос 1:
        !          1862: Героиня Набокова, опустив после сложного разговора телефонную трубку,
        !          1863: прислушалась и с облегчением услышала только ЕГО. Назовите ЕГО тремя
        !          1864: словами, начинающимися на одну и ту же букву.
        !          1865: 
        !          1866: Ответ:
        !          1867: Стук своего сердца.
        !          1868: 
        !          1869: Зачет:
        !          1870: Стучание своего сердца; стук/стучание собственного сердца.
        !          1871: 
        !          1872: Комментарий:
        !          1873: Она жила в большом доме и боялась, что этот разговор могли подслушать.
        !          1874: Рядом никого не оказалось. Это не "так судьба стучится в двери", конечно
        !          1875: же, но всё же.
        !          1876: 
        !          1877: Источник:
        !          1878: В.В. Набоков. Защита Лужина. http://www.flibusta.is/b/385732/read
        !          1879: 
        !          1880: Автор:
        !          1881: Людмила Полякова (Харьков)
        !          1882: 
        !          1883: Вопрос 2:
        !          1884: [Ведущему: внимательно прочитать конец первого предложения, чтобы
        !          1885: команды четко уяснили грамматическую структуру.]
        !          1886:    В одном фильме пилигрим сообщает, что первым по доблести был Ричард
        !          1887: Львиное Сердце, а ПРОПУСК. Уже в конце апреля 2016 года стало понятно,
        !          1888: что, несмотря на неверие букмекеров еще осенью, ПРОПУСК - и это как
        !          1889: минимум. Пропуски на слух неотличимы. Заполните любой из них двумя
        !          1890: словами.
        !          1891: 
        !          1892: Ответ:
        !          1893: Лестер второй.
        !          1894: 
        !          1895: Зачет:
        !          1896: "Лестер" - второй.
        !          1897: 
        !          1898: Комментарий:
        !          1899: Вторым был благородный граф Лестер.
        !          1900: 
        !          1901: Источник:
        !          1902: Х/ф "Баллада о доблестном рыцаре Айвенго" (1982), реж. Сергей Тарасов,
        !          1903: 8-я минута.
        !          1904: 
        !          1905: Автор:
        !          1906: Александр Голиков (Харьков)
        !          1907: 
        !          1908: Вопрос 3:
        !          1909: [Ведущему: четко прочитать "в Шуне" и "в Кване".]
        !          1910:    В самом начале пьесы один из персонажей саркастично упоминает
        !          1911: случайность в Шуне и случайность в Кване. Впрочем, им все-таки больше
        !          1912: везет... Где?
        !          1913: 
        !          1914: Ответ:
        !          1915: В Сычуани.
        !          1916: 
        !          1917: Зачет:
        !          1918: В Сезуане.
        !          1919: 
        !          1920: Комментарий:
        !          1921: Боги в произведении Бертольта Брехта уже не верят, что им удастся найти
        !          1922: хотя бы одного доброго человека, но наконец-то нашелся хотя бы один - в
        !          1923: городе Сычуань.
        !          1924: 
        !          1925: Источник:
        !          1926: Б. Брехт. Добрый человек из Сычуани.
        !          1927: http://www.flibusta.is/b/469976/read
        !          1928: 
        !          1929: Автор:
        !          1930: Александр Голиков (Харьков)
        !          1931: 
        !          1932: Вопрос 4:
        !          1933: Когда брехтовские боги обсуждают водоноса, один из них обвиняет его в
        !          1934: мошенничестве, держа в руках кружку, из которой водонос их напоил водой.
        !          1935: Всё дело в НЕМ. Назовите ЕГО двумя словами, начинающимися на одну и ту
        !          1936: же букву.
        !          1937: 
        !          1938: Ответ:
        !          1939: Двойное дно.
        !          1940: 
        !          1941: Комментарий:
        !          1942: Водонос дал богам напиться воды из кружки с двойным дном, которое
        !          1943: символизирует нечестность и мошенничество.
        !          1944: 
        !          1945: Источник:
        !          1946: Б. Брехт. Добрый человек из Сычуани.
        !          1947: http://www.flibusta.is/b/469976/read
        !          1948: 
        !          1949: Автор:
        !          1950: Александр Голиков (Харьков)
        !          1951: 
        !          1952: Вопрос 5:
        !          1953: Внимание, в одном из слов вопроса мы пропустили две буквы.
        !          1954:    Вильфредо Парето - итальянский социолог-консерватор, который,
        !          1955: несмотря на свой аристократизм по происхождению, пережил семейную драму.
        !          1956: Преподаватель социологии, рассказывая об этом, заявил, что его жена,
        !          1957: Александра Бакунина, изменила ему с маркизом. Восстановите исходное
        !          1958: слово.
        !          1959: 
        !          1960: Ответ:
        !          1961: Марксизмом.
        !          1962: 
        !          1963: Комментарий:
        !          1964: Фамилия Александры Бакуниной явно указывает на ее марксистские и
        !          1965: анархистские контакты.
        !          1966: 
        !          1967: Источник:
        !          1968:    1. ЛНА на лекции.
        !          1969:    2. http://50.economicus.ru/index.php?ch=5&le=42&r=4&z=1
        !          1970:    3. http://ru.wikipedia.org/wiki/Парето,_Вильфредо
        !          1971: 
        !          1972: Автор:
        !          1973: Александр Голиков (Харьков)
        !          1974: 
        !          1975: Вопрос 6:
        !          1976: В мае 1949 года ОН передал свой список из 38 журналистов и писателей
        !          1977: спецслужбам. В этом списке были такие характеристики, как "бесчестный
        !          1978: карьерист", "сентиментальный симпатизант", "еврейский",
        !          1979: "антибританский", "хорошо нажился в СССР", "очень антибелый" и тому
        !          1980: подобное. Назовите ЕГО.
        !          1981: 
        !          1982: Ответ:
        !          1983: [Эрик] Блэр.
        !          1984: 
        !          1985: Зачет:
        !          1986: [Джордж] Оруэлл.
        !          1987: 
        !          1988: Комментарий:
        !          1989: Да, марксист по убеждениям Эрик Блэр работал на Большого Брата.
        !          1990: 
        !          1991: Источник:
        !          1992: http://gefter.ru/archive/15452
        !          1993: 
        !          1994: Автор:
        !          1995: Александр Голиков (Харьков)
        !          1996: 
        !          1997: Вопрос 7:
        !          1998: Согласно одному несерьезному источнику, в результате забастовки цыган
        !          1999: тысячи горожан остались ПРОПУСК. По мнению профессора Криса Синхи,
        !          2000: представители племени амондава живут ПРОПУСК, поэтому могут говорить
        !          2001: только об одном событии. Какие два слова на одну и ту же букву мы дважды
        !          2002: пропустили?
        !          2003: 
        !          2004: Ответ:
        !          2005: Без будущего.
        !          2006: 
        !          2007: Комментарий:
        !          2008: Вряд ли это племя живет в мире Оруэлла, но будущего времени в его языке
        !          2009: действительно нет.
        !          2010: 
        !          2011: Источник:
        !          2012:    1. http://www.anekdot.ru/id/256514/
        !          2013:    2. http://www.bbc.com/russian/international/2011/05/110520_amondawa_language_lacks_time.shtml
        !          2014: 
        !          2015: Автор:
        !          2016: Людмила Полякова (Харьков)
        !          2017: 
        !          2018: Вопрос 8:
        !          2019:    <раздатка>
        !          2020:    leaned
        !          2021:    </раздатка>
        !          2022:    Седьмой эпизод "Звездных войн" многие обвиняют во вторичности. В
        !          2023: неологизме, который газета "Вашингтон Пост" применяет по отношению к
        !          2024: одной из планет в этом фильме, мы убрали четыре буквы. Восстановите
        !          2025: исходное слово.
        !          2026: 
        !          2027: Ответ:
        !          2028: Alderaaned.
        !          2029: 
        !          2030: Комментарий:
        !          2031: Планета была уничтожена, как и Алдераан, причем как способ уничтожения,
        !          2032: так и кинокартинка дают основания подозревать создателей фильма во
        !          2033: вторичности.
        !          2034: 
        !          2035: Источник:
        !          2036: https://www.washingtonpost.com/news/monkey-cage/wp/2016/01/06/here-are-three-ways-that-star-wars-is-a-neoconservative-universe/
        !          2037: 
        !          2038: Автор:
        !          2039: Людмила Полякова (Харьков)
        !          2040: 
        !          2041: Вопрос 9:
        !          2042: Как утверждают журналисты, город Вольфсбург подхватывает воспаление
        !          2043: легких, когда корпорация "Фольксваген" ДЕЛАЕТ ЭТО. Желая показать, что
        !          2044: он опознал фрагменты плагиата, Джоакино Россини на концерте неоднократно
        !          2045: ДЕЛАЛ ЭТО. Что делал?
        !          2046: 
        !          2047: Ответ:
        !          2048: Снимал шляпу.
        !          2049: 
        !          2050: Комментарий:
        !          2051: От "Фольксвагена" сильно зависит благосостояние Вольфсбурга. А Россини
        !          2052: "здоровался" со "встреченными" им во время концерта коллегами.
        !          2053: 
        !          2054: Источник:
        !          2055:    1. "Футбол", 2015, N 66 (1352). - С. 21.
        !          2056:    2. А.И. Муха. Музыканты смеются. http://www.flibusta.is/b/273591/read
        !          2057: 
        !          2058: Автор:
        !          2059: Александр Голиков (Харьков)
        !          2060: 
        !          2061: Вопрос 10:
        !          2062: В книге "Алиса и Алисия" положительный персонаж Алиса Селезнёва
        !          2063: приезжает на Суматру за одним экземпляром, путешествует во времени и
        !          2064: знакомится с диктатором и тираном Алисией I. Какое слово мы пропустили в
        !          2065: предыдущем предложении?
        !          2066: 
        !          2067: Ответ:
        !          2068: Бабочки.
        !          2069: 
        !          2070: Комментарий:
        !          2071: Как можно понять из имени, Алисия - негативное развитие Алисы.
        !          2072: Получается своеобразный эффект бабочки.
        !          2073: 
        !          2074: Источник:
        !          2075: К. Булычёв. Алиса и Алисия. http://www.flibusta.is/b/108182/read
        !          2076: 
        !          2077: Автор:
        !          2078: Александр Голиков (Харьков)
        !          2079: 
        !          2080: Вопрос 11:
        !          2081: Известный персонаж книги Бориса Акунина при виде Тенерифе произносит,
        !          2082: как это ни странно для русскоязычного читателя, короткое слово. Назовите
        !          2083: это двусложное слово.
        !          2084: 
        !          2085: Ответ:
        !          2086: Яма.
        !          2087: 
        !          2088: Комментарий:
        !          2089: Тенерифе - это остров со знаменитым вулканом Тейде. Персонаж, Масахиро
        !          2090: Сибата, - японец, слуга Фандорина, и для него это "яма" (сравните с
        !          2091: "Фудзи-яма", "Тате-яма").
        !          2092: 
        !          2093: Источник:
        !          2094:    1. Б. Акунин. Планета Вода. http://www.flibusta.is/b/401666/read
        !          2095:    2. http://ru.wikipedia.org/wiki/Тенерифе
        !          2096:    3. http://ru.wikipedia.org/wiki/Фудзияма
        !          2097:    4. http://ru.wikipedia.org/wiki/Татеяма_(гора)
        !          2098: 
        !          2099: Автор:
        !          2100: Людмила Полякова (Харьков)
        !          2101: 
        !          2102: Вопрос 12:
        !          2103: В постмодернистской пародии на пьесу "Ричард III" Эдвард расстраивается,
        !          2104: что его после смерти не слушают, и упоминает о планах переезда. Ответьте
        !          2105: абсолютно точно: куда именно?
        !          2106: 
        !          2107: Ответ:
        !          2108: Эльсинор.
        !          2109: 
        !          2110: Зачет:
        !          2111: Замок Эльсинор.
        !          2112: 
        !          2113: Комментарий:
        !          2114: И, как и подобает в постмодернистском тексте, Эдвард становится тенью -
        !          2115: видимо, тенью отца Гамлета.
        !          2116: 
        !          2117: Источник:
        !          2118: http://the-mockturtle.livejournal.com/770332.html
        !          2119: 
        !          2120: Автор:
        !          2121: Людмила Полякова (Харьков)
        !          2122: 
        !          2123: Тур:
        !          2124: Третий игровой день. 3 тур
        !          2125: 
        !          2126: Дата:
        !          2127: 09-Dec-2016
        !          2128: 
        !          2129: Редактор:
        !          2130: Олег Михеев и Александр Усков (Краснодар)
        !          2131: 
        !          2132: Инфо:
        !          2133: Редакторы благодарят за тестирование вопросов и ценные советы: Андрея
        !          2134: Баландина и Андрея Гречишникова (оба - Москва), Максима Карачуна
        !          2135: (Краснодар), Маргариту Кирюшину и Инессу Кличманову (обе - Москва), Илью
        !          2136: Кукушкина и Дениса Лагутина (оба - Краснодар), Наталью Мойсик
        !          2137: (Ростов-на-Дону), Марию Подрядчикову (Волгоград), Павла Солюкова
        !          2138: (Краснодар), Дмитрия Тарарыкова и Артема Шинкевича (оба - Москва).
        !          2139: 
        !          2140: Вопрос 1:
        !          2141: По легенде, художник ЗЕвксис не смог найти достаточно красивую девушку,
        !          2142: для того чтобы изобразить Елену Прекрасную. Мэтью ГАмперт сравнивает
        !          2143: ЗЕвксиса с заглавным героем произведения XIX века. Назовите этого героя.
        !          2144: 
        !          2145: Ответ:
        !          2146: [Виктор] Франкенштейн.
        !          2147: 
        !          2148: Комментарий:
        !          2149: Легенда гласит, что ЗЕвксис отобрал для позирования пятерых девушек и
        !          2150: позаимствовал лучшие черты каждой из них для портрета Елены. В романе
        !          2151: Мэри Шелли Франкенштейн создал своего монстра, используя различные
        !          2152: фрагменты тел.
        !          2153: 
        !          2154: Источник:
        !          2155:    1. Matthew Gumpert. Grafting Helen: The Abduction of the Classical
        !          2156: Past.
        !          2157: https://books.google.ru/books?id=5TM1hIAfX-EC&pg=PA258#v=onepage&q&f=false
        !          2158:    2. http://ru.wikipedia.org/wiki/Виктор_Франкенштейн
        !          2159: 
        !          2160: Автор:
        !          2161: Олег Михеев (Краснодар)
        !          2162: 
        !          2163: Вопрос 2:
        !          2164: Желающие поучаствовать в проводимом в Сахаре марафоне должны оплатить
        !          2165: вступительный взнос. В русское специализированное название того, для
        !          2166: чего, помимо прочего, предназначен этот взнос, входит число. Напишите
        !          2167: это число.
        !          2168: 
        !          2169: Ответ:
        !          2170: 200.
        !          2171: 
        !          2172: Комментарий:
        !          2173: Марафон является достаточно опасным, так что этот взнос предназначен на
        !          2174: случай смерти участника и репатриации его тела на родину. В вооруженных
        !          2175: силах тела погибших принято называть "грузом 200".
        !          2176: 
        !          2177: Источник:
        !          2178:    1. https://www.si.com/vault/2002/04/29/8103066/hot--cold-at-the-marathon-des-sables-billed-as-the-worlds-toughest-footrace-even-the-stoutest-of-competitors-couldnt-avoid-getting-saharan-sand-kicked-and-blown-in-their-faces
        !          2179:    2. http://ru.wikipedia.org/wiki/Груз_200
        !          2180: 
        !          2181: Автор:
        !          2182: Олег Михеев (Краснодар)
        !          2183: 
        !          2184: Вопрос 3:
        !          2185: Рассказывают, что каждый день с февраля 1858 года на протяжении
        !          2186: четырнадцати лет Бобби можно было встретить на кладбище ГрейфрАйерс в
        !          2187: Эдинбурге. В статье журнала "National Geographic" [нэшнл джиогрЭфик]
        !          2188: Бобби называют предшественником... Кого?
        !          2189: 
        !          2190: Ответ:
        !          2191: Хатико.
        !          2192: 
        !          2193: Комментарий:
        !          2194: Скайтерьер Бобби стал знаменит тем, что охранял могилу хозяина, а затем
        !          2195: был похоронен недалеко от ворот кладбища. Вскоре в Эдинбурге был открыт
        !          2196: памятник Бобби. Более известна похожая история про японского пса Хатико.
        !          2197: 
        !          2198: Источник:
        !          2199: http://www.nat-geo.ru/fact/38617-predannyy-pes-bobbi/
        !          2200: 
        !          2201: Автор:
        !          2202: Александр Усков (Краснодар)
        !          2203: 
        !          2204: Вопрос 4:
        !          2205: По одной из версий, альбом известной группы был назван в честь команды
        !          2206: для устройств Macintosh [макинтОш]. Сейчас схожая по структуре команда
        !          2207: используется в ИКСЕ. На вышеупомянутом альбоме ИКС тоже есть. Назовите
        !          2208: ИКС одним словом.
        !          2209: 
        !          2210: Ответ:
        !          2211: Android.
        !          2212: 
        !          2213: Зачет:
        !          2214: Андроид.
        !          2215: 
        !          2216: Комментарий:
        !          2217: Фраза "OK Computer", ставшая названием альбомы группы "Radiohead"
        !          2218: [рэйдиохЭд], была командой для голосового распознавания в Macintosh. В
        !          2219: операционной системе Android для той же цели используется команда "OK
        !          2220: Google". Композиция "Paranoid Android" [паранОид андрОид] входит в
        !          2221: вышеупомянутый альбом.
        !          2222: 
        !          2223: Источник:
        !          2224: http://ru.wikipedia.org/wiki/OK_Computer
        !          2225: 
        !          2226: Автор:
        !          2227: Александр Усков (Краснодар)
        !          2228: 
        !          2229: Вопрос 5:
        !          2230: По официальной версии, в Краснодаре весной этого года ОНО не состоялось
        !          2231: по соображениям безопасности. Кроме того, представитель епархии упомянул
        !          2232: негативные ассоциации с древним кельтским обычаем. Назовите ЕГО.
        !          2233: 
        !          2234: Ответ:
        !          2235: Сожжение [чучела] Масленицы.
        !          2236: 
        !          2237: Зачет:
        !          2238: Сжигание [чучела] Масленицы.
        !          2239: 
        !          2240: Комментарий:
        !          2241: Как один из ближайших аналогов русского обычая, представители епархии
        !          2242: упомянули обряд жертвоприношения друидов, в процессе которого жертва
        !          2243: заключается в специальную клетку, называемую плетеным человеком, и
        !          2244: подвергается сожжению.
        !          2245: 
        !          2246: Источник:
        !          2247:    1. https://www.yuga.ru/news/392580/
        !          2248:    2. http://ru.wikipedia.org/wiki/Плетёный_человек
        !          2249: 
        !          2250: Автор:
        !          2251: Александр Усков (Краснодар)
        !          2252: 
        !          2253: Вопрос 6:
        !          2254: В этом вопросе словом "ИКС" мы заменили другое слово.
        !          2255:    Одна из статей американского закона устанавливает, в каких случаях
        !          2256: кража считается совершённой в крупном размере. Например, если сумма
        !          2257: украденного превышает 950 долларов или если украден ИКС. С этой статьей
        !          2258: связывают происхождение фразы из трех слов, зачастую сокращаемой до
        !          2259: аббревиатуры. Назовите эту аббревиатуру.
        !          2260: 
        !          2261: Ответ:
        !          2262: GTA.
        !          2263: 
        !          2264: Зачет:
        !          2265: ГТА.
        !          2266: 
        !          2267: Комментарий:
        !          2268: Выражение, которым названа популярная компьютерная игра, появилось еще в
        !          2269: середине века. Согласно законам Калифорнии, одним из условий, когда
        !          2270: кража считается крупной, является угон автомобиля, вне зависимости от
        !          2271: его цены. Так в уголовный жаргон вошло выражение "Grand Theft Auto"
        !          2272: [гранд зэфт Ото]. Этим, кстати, объясняется и ненатуральный для
        !          2273: английского языка порядок слов, ведь более правильным и привычным было
        !          2274: бы "Auto Grand Theft".
        !          2275: 
        !          2276: Источник:
        !          2277: http://leginfo.legislature.ca.gov/faces/codes_displaySection.xhtml?lawCode=PEN&sectionNum=487
        !          2278: 
        !          2279: Автор:
        !          2280: Олег Михеев (Краснодар)
        !          2281: 
        !          2282: Вопрос 7:
        !          2283: Согласно не вполне серьезному совету, британцам не стоит покупать
        !          2284: красные автомобили. Дело в том, что заскучавшие полицейские иногда
        !          2285: соревнуются в так называемой "дорожной" разновидности... Ответьте словом
        !          2286: английского происхождения: разновидности чего?
        !          2287: 
        !          2288: Ответ:
        !          2289: Снукер.
        !          2290: 
        !          2291: Комментарий:
        !          2292: Снукер - это разновидность бильярда, в который играют 15 красными шарами
        !          2293: и 6 шарами других цветов, причем забиваться красные и цветные шары
        !          2294: должны поочередно. Популярная в Англии городская легенда гласит, что
        !          2295: полицейские иногда играют в снукер машинами, т.е. останавливают машины в
        !          2296: соответствии с их цветом и правилами снукера, поэтому водители красных
        !          2297: автомобилей имеют наибольший шанс быть остановленными.
        !          2298: 
        !          2299: Источник:
        !          2300:    1. http://www.exposedpolice.com/uk-traffic-police-playing-motorway-snooker/
        !          2301:    2. http://ru.wikipedia.org/wiki/Снукер
        !          2302: 
        !          2303: Автор:
        !          2304: Олег Михеев (Краснодар)
        !          2305: 
        !          2306: Вопрос 8:
        !          2307: По одной из версий, причиной тяжелых последствий происшествия в Альпах
        !          2308: могла стать установленная на НЕМ камера. На НЕМ можно было увидеть семь
        !          2309: звезд. Назовите ЕГО двумя словами, начинающимися на одну и ту же букву.
        !          2310: 
        !          2311: Ответ:
        !          2312: Шлем Шумахера.
        !          2313: 
        !          2314: Комментарий:
        !          2315: В 2013 году Михаэль Шумахер во время катания на лыжах упал и получил
        !          2316: серьезную травму головы. Существует версия, что причиной этого могла
        !          2317: стать камера GoPro [гОу про], установленная на шлеме и разбившая его при
        !          2318: ударе. Согласно распространенному заблуждению, семь звезд на гоночном
        !          2319: шлеме Шумахера олицетворяют семь чемпионских титулов "Формулы-1", но на
        !          2320: самом деле такой дизайн шлема был у него практически с начала карьеры.
        !          2321: 
        !          2322: Источник:
        !          2323:    1. http://www.telegraph.co.uk/sport/motorsport/formulaone/michael-schumacher/10640839/Michael-Schumacher-skiing-crash-did-helmet-camera-cause-head-injuries.html
        !          2324:    2. http://www.alamy.com/stock-photo-michael-schumacher-a-racing-helmet-of-the-seven-time-formula-1-world-31707308.html
        !          2325: 
        !          2326: Автор:
        !          2327: Олег Михеев (Краснодар)
        !          2328: 
        !          2329: Вопрос 9:
        !          2330: Согласно одному шуточному изображению, ученому удалось создать видимость
        !          2331: того, что он всё еще работает над НЕЙ. Однако очевидно, что работа уже
        !          2332: завершена. Назовите ЕЕ словом греческого происхождения.
        !          2333: 
        !          2334: Ответ:
        !          2335: Голограмма.
        !          2336: 
        !          2337: Зачет:
        !          2338: Голография.
        !          2339: 
        !          2340: Комментарий:
        !          2341: Ученый создал голограмму самого себя, которая изображает его за работой.
        !          2342: 
        !          2343: Источник:
        !          2344: http://www.ifunny.com/pictures/greg-was-still-hard-work-hologram-technology/
        !          2345: 
        !          2346: Автор:
        !          2347: Александр Усков (Краснодар)
        !          2348: 
        !          2349: Вопрос 10:
        !          2350: В начале XX века Старый Том помогал китобоям в охоте, взамен получая
        !          2351: часть добычи. В одной статье это сотрудничество охарактеризовано
        !          2352: термином, впервые употребленным в современном значении в 1940 году.
        !          2353: Назовите этот термин словом с удвоенной согласной.
        !          2354: 
        !          2355: Ответ:
        !          2356: Коллаборационизм.
        !          2357: 
        !          2358: Комментарий:
        !          2359: Старый Том был вожаком стаи косаток, сопровождавших усатых китов в
        !          2360: бухту, где охотились австралийские китобои. Иногда косатки даже хватали
        !          2361: зубами веревку гарпуна и помогали в буксировке; взамен китобои оставляли
        !          2362: им языки убитых китов. Коллаборационизм - это осознанное и добровольное
        !          2363: сотрудничество с врагом. Впервые так стали называть сотрудничавшее с
        !          2364: немцами правительство ВишИ во Франции.
        !          2365: 
        !          2366: Источник:
        !          2367:    1. http://www.jewishhistoryaustralia.net/Nulla_Nulla_Story/A_5_Killers_in_Eden.htm
        !          2368:    2. http://en.wikipedia.org/wiki/Collaborationism#Etymology
        !          2369: 
        !          2370: Автор:
        !          2371: Олег Михеев (Краснодар)
        !          2372: 
        !          2373: Вопрос 11:
        !          2374: Журналист и критик Алекс Шепард известен скептическими высказываниями.
        !          2375: Одно из них привело к тому, что в октябре этого года Шепард опубликовал
        !          2376: фотографию пластинки Боба Дилана, на которой лежат два металлических
        !          2377: предмета. Назовите эти предметы.
        !          2378: 
        !          2379: Ответ:
        !          2380: Вилка, нож.
        !          2381: 
        !          2382: Зачет:
        !          2383: В любом порядке.
        !          2384: 
        !          2385: Комментарий:
        !          2386: Боб Дилан регулярно появлялся в списке фаворитов на получение
        !          2387: Нобелевской премии по литературе. Несмотря на это, Алекс Шепард
        !          2388: неоднократно заявлял, что не верит в победу Дилана, а в 2015 году даже
        !          2389: пообещал съесть свой экземпляр пластинки "Blood on the Tracks" [блад он
        !          2390: зэ трэкс], если это всё же случится.
        !          2391: 
        !          2392: Источник:
        !          2393:    1. https://twitter.com/alex_shephard/status/786530758571724800/
        !          2394:    2. https://newrepublic.com/article/123058/who-will-win-nobel-prize-literature
        !          2395: 
        !          2396: Автор:
        !          2397: Александр Усков (Краснодар)
        !          2398: 
        !          2399: Вопрос 12:
        !          2400: Проведя раскопки в одном американском городе на улице КонтИ, ученые
        !          2401: обнаружили, в частности, много румян и бутылок из-под ликера. Это
        !          2402: позволило предположить, что заведение, упоминаемое в известном
        !          2403: произведении, действительно существовало. Назовите это произведение.
        !          2404: 
        !          2405: Ответ:
        !          2406: "The House of the Rising Sun" [чтецу: зэ хАус оф зэ рАйзин сан].
        !          2407: 
        !          2408: Зачет:
        !          2409: Дом восходящего солнца.
        !          2410: 
        !          2411: Комментарий:
        !          2412: "Дом восходящего солнца" - народная американская песня. Обычно под ним
        !          2413: понимают то ли тюрьму, то ли бордель середины XIX века в Новом Орлеане.
        !          2414: Существовало ли это заведение, до сих пор остается открытым вопросом. По
        !          2415: словам археолога Шэннон Доуди, проведшей раскопки на предполагаемом
        !          2416: месте, известном из газет того времени, обстановка выглядела похожей на
        !          2417: бордель.
        !          2418: 
        !          2419: Источник:
        !          2420:    1. http://en.wikipedia.org/wiki/The_House_of_the_Rising_Sun
        !          2421:    2. http://www.angelpig.com/house_history.html
        !          2422: 
        !          2423: Автор:
        !          2424: Олег Михеев (Краснодар)
        !          2425: 
        !          2426: Тур:
        !          2427: Финал (Минск). 1 тур
        !          2428: 
        !          2429: Дата:
        !          2430: 25-Mar-2017
        !          2431: 
        !          2432: Редактор:
        !          2433: Ольга Кузьма и Андрей Кузьма (Санкт-Петербург)
        !          2434: 
        !          2435: Инфо:
        !          2436: Редакторы тура благодарят за помощь и ценные замечания Алексея Рабина,
        !          2437: Максима Веслополова, Евгения и Аллу Муштай (все - Санкт-Петербург) и
        !          2438: Дмитрия Свинтицкого (Могилев).
        !          2439: 
        !          2440: Вопрос 1:
        !          2441: Из первоначального списка претендентов на звание "музыкальный символ
        !          2442: чемпионата мира - 2018" сразу были вычеркнуты все духовые инструменты.
        !          2443: Причиной такого решения стали ОНИ. Назовите ИХ иностранным словом.
        !          2444: 
        !          2445: Ответ:
        !          2446: Вувузелы.
        !          2447: 
        !          2448: Комментарий:
        !          2449: Вувузелы - музыкальный символ чемпионата мира по футболу 2010 года -
        !          2450: многие до сих пор вспоминают с ужасом. "Вувузела" в переводе с
        !          2451: зулусского - "делать шум". Музыкальным символом ЧМ-2018 стали ложки.
        !          2452: 
        !          2453: Источник:
        !          2454:    1. http://radiozenit.ru/news_full/uid/621
        !          2455:    2. http://ru.wikipedia.org/wiki/Вувузела
        !          2456: 
        !          2457: Автор:
        !          2458: Ольга Кузьма, Андрей Кузьма (Санкт-Петербург)
        !          2459: 
        !          2460: Вопрос 2:
        !          2461: Уильяма Фридмана, пытавшегося предупредить ВМФ США о налёте на
        !          2462: Пёрл-Харбор, называют американским ИМ. ОН родился в 1895 году в
        !          2463: Бакинской губернии. Назовите ЕГО.
        !          2464: 
        !          2465: Ответ:
        !          2466: [Рихард] Зорге.
        !          2467: 
        !          2468: Источник:
        !          2469: Станислав Зигуненко. 100 великих загадок истории флота. - М.: Вече,
        !          2470: 2012. - С. 223.
        !          2471: 
        !          2472: Автор:
        !          2473: Ольга Кузьма, Андрей Кузьма (Санкт-Петербург)
        !          2474: 
        !          2475: Вопрос 3:
        !          2476: Адель Алексеева пишет, что на рисунке Пушкина, в отличие от работ многих
        !          2477: художников, ОН не смешной, а скорее благородный, породистый. Назовите
        !          2478: ЕГО двумя словами, не используя кавычки.
        !          2479: 
        !          2480: Ответ:
        !          2481: Нос Гоголя.
        !          2482: 
        !          2483: Источник:
        !          2484: Адель Алексеева. Как влюблялись, творили и шалили наши классики. - М.:
        !          2485: Вече, 2016. - С. 53.
        !          2486: 
        !          2487: Автор:
        !          2488: Ольга Кузьма, Андрей Кузьма (Санкт-Петербург)
        !          2489: 
        !          2490: Вопрос 4:
        !          2491: Известный журналист полагал, что будет похоронен рядом с человеком, чей
        !          2492: девиз - "Христианство, торговля и цивилизация", но настоятель
        !          2493: Вестминстерского аббатства воспротивился этому. Назовите фамилию этого
        !          2494: журналиста.
        !          2495: 
        !          2496: Ответ:
        !          2497: Стэнли.
        !          2498: 
        !          2499: Комментарий:
        !          2500: Слово "полагал" - подсказка. Генри Мортон Стэнли нашел пропавшего в
        !          2501: Африке Давида Ливингстона и приветствовал его словами "Доктор
        !          2502: Ливингстон, полагаю?".
        !          2503: 
        !          2504: Источник:
        !          2505:    1. Великие путешествия. Герои, покорившие планету. - СПб.:
        !          2506: "Пресс-курьер", N 14 за 2016 год. - С. 104.
        !          2507:    2. http://ru.wikipedia.org/wiki/Ливингстон,_Давид
        !          2508: 
        !          2509: Автор:
        !          2510: Ольга Кузьма, Андрей Кузьма (Санкт-Петербург)
        !          2511: 
        !          2512: Вопрос 5:
        !          2513: Болельщики "Манчестер Юнайтед" дали лидерам своего клуба Полю Погба,
        !          2514: Златану Ибрагимовичу и Уэйну Руни очень короткое прозвище, намекающее на
        !          2515: фатальность действий данных футболистов для соперников. Напишите это
        !          2516: прозвище по-английски.
        !          2517: 
        !          2518: Ответ:
        !          2519: RIP.
        !          2520: 
        !          2521: Комментарий:
        !          2522: Прозвище образовано по первым буквам фамилий игроков. RIP - сокращение
        !          2523: от "rest in peace" [рест ин пис]. "Покойся с миром" - традиционная
        !          2524: надпись на надгробьях в англоязычных странах.
        !          2525: 
        !          2526: Источник:
        !          2527: https://www.championat.com/football/news-2543880-trojke-igrokov-mju-runi-ibragimovich-pogba-dali-shutlivoe-prozvische-r-i-p.html
        !          2528: 
        !          2529: Автор:
        !          2530: Ольга Кузьма, Андрей Кузьма (Санкт-Петербург)
        !          2531: 
        !          2532: Вопрос 6:
        !          2533: На Каспии издавна было принято расстилать в прибрежной полосе тряпки и
        !          2534: старые ковры, прижимая их камнями. Таким образом местные жители ДЕЛАЛИ
        !          2535: ЭТО для домашнего использования. Россия входят в тройку лидеров среди
        !          2536: стран, ДЕЛАЮЩИХ ЭТО. Какие слова мы заменили словами "ДЕЛАТЬ ЭТО"?
        !          2537: 
        !          2538: Ответ:
        !          2539: Добывать нефть.
        !          2540: 
        !          2541: Источник:
        !          2542: Станислав Зигуненко. 100 великих загадок истории флота. - М.: Вече,
        !          2543: 2012. - С. 157.
        !          2544: 
        !          2545: Автор:
        !          2546: Ольга Кузьма, Андрей Кузьма (Санкт-Петербург)
        !          2547: 
        !          2548: Вопрос 7:
        !          2549: В вопросе есть замена.
        !          2550:    Известный гурман Александр Дюма писал: "Я немного поохотился на
        !          2551: берегах Каспия, где в таком же изобилии водятся дикие гуси, утки,
        !          2552: пеликаны, как на Сене - собаки". Восстановите замененное нами слово.
        !          2553: 
        !          2554: Ответ:
        !          2555: Лягушки.
        !          2556: 
        !          2557: Источник:
        !          2558: Великие путешествия. Герои, покорившие планету. - СПб.: Пресс-курьер, N
        !          2559: 14 за 2016 год. - С. 141.
        !          2560: 
        !          2561: Автор:
        !          2562: Ольга Кузьма, Андрей Кузьма (Санкт-Петербург)
        !          2563: 
        !          2564: Вопрос 8:
        !          2565: Майкл Крайтон пишет, что в период правления бельгийской колониальной
        !          2566: администрации случаи каннибализма стали более редкими, а к началу 1960-х
        !          2567: годов в Конго даже появилось несколько ИХ. Одно из НИХ фигурирует в
        !          2568: заглавии романа 1983 года. Назовите автора этого романа.
        !          2569: 
        !          2570: Ответ:
        !          2571: [Стивен] Кинг.
        !          2572: 
        !          2573: Комментарий:
        !          2574: ОНИ - кладбища. Роман - "Кладбище домашних животных".
        !          2575: 
        !          2576: Источник:
        !          2577:    1. М. Крайтон. Конго. http://flibusta.is/b/135768/read
        !          2578:    2. http://ru.wikipedia.org/wiki/Кладбище_домашних_животных_(роман)
        !          2579: 
        !          2580: Автор:
        !          2581: Ольга Кузьма, Андрей Кузьма (Санкт-Петербург)
        !          2582: 
        !          2583: Вопрос 9:
        !          2584: Для примата галаго из семейства лориобразных слух не менее важен, чем
        !          2585: зрение. За внешний вид галаго часто называют именем персонажа,
        !          2586: появившегося около полувека назад. Назовите этого персонажа.
        !          2587: 
        !          2588: Ответ:
        !          2589: Чебурашка.
        !          2590: 
        !          2591: Комментарий:
        !          2592: У Галаго не только глаза, но и уши велики.
        !          2593: 
        !          2594: Источник:
        !          2595: Дмитрий Бердышев. Самые необычные животные. - М.: ЭНАС-КНИГА, 2016. - С.
        !          2596: 97, 101.
        !          2597: 
        !          2598: Автор:
        !          2599: Ольга Кузьма, Андрей Кузьма (Санкт-Петербург)
        !          2600: 
        !          2601: Вопрос 10:
        !          2602: Однажды Роберт Вуд растопил несколько килограммов свинца и вылил их в
        !          2603: небольшое отверстие в земле. После того как свинец застыл, Вуд, потратив
        !          2604: на раскопки несколько часов, извлек его, получив своеобразный
        !          2605: скульптурный портрет ЕЕ. Назовите ЕЕ.
        !          2606: 
        !          2607: Ответ:
        !          2608: Молния.
        !          2609: 
        !          2610: Комментарий:
        !          2611: Разветвленная отливка ушла в землю на глубину более трех метров.
        !          2612: 
        !          2613: Источник:
        !          2614: З. Столбовский. Великие тайны и загадки мира. Опасности и угрозы. - М.:
        !          2615: Мартин, 2005. - С. 156.
        !          2616: 
        !          2617: Автор:
        !          2618: Ольга Кузьма, Андрей Кузьма (Санкт-Петербург)
        !          2619: 
        !          2620: Вопрос 11:
        !          2621: 26 апреля 1945 года Николай Масалов спас ребенка. Через несколько дней в
        !          2622: его полк приехал человек, поговоривший с Николаем и сделавший несколько
        !          2623: набросков его внешности. Назовите этого человека.
        !          2624: 
        !          2625: Ответ:
        !          2626: [Евгений] Вучетич.
        !          2627: 
        !          2628: Комментарий:
        !          2629: Впоследствии Вучетич создал знаменитый памятник, установленный в
        !          2630: Трептов-парке.
        !          2631: 
        !          2632: Источник:
        !          2633: Вячеслав Бондаренко. 100 великих подвигов России. - М.: Вече, 2014. - С.
        !          2634: 306.
        !          2635: 
        !          2636: Автор:
        !          2637: Ольга Кузьма, Андрей Кузьма (Санкт-Петербург)
        !          2638: 
        !          2639: Вопрос 12:
        !          2640: В 1966 году отряд советских подводных лодок совершил кругосветное
        !          2641: путешествие, не всплывая на поверхность. За время плавания советские
        !          2642: моряки за ненадобностью ни разу не совершили некое действие. Шесть лет
        !          2643: назад в Белоруссии это действие сочли ненужным. Назовите ЕГО.
        !          2644: 
        !          2645: Ответ:
        !          2646: Перевод часов.
        !          2647: 
        !          2648: Источник:
        !          2649:    1. Станислав Зигуненко. 100 великих загадок истории флота. - М.:
        !          2650: Вече, 2012. - С. 246.
        !          2651:    2. http://ru.wikipedia.org/wiki/Время_в_Белоруссии
        !          2652: 
        !          2653: Автор:
        !          2654: Ольга Кузьма, Андрей Кузьма (Санкт-Петербург)
        !          2655: 
        !          2656: Тур:
        !          2657: Финал (Минск). 2 тур
        !          2658: 
        !          2659: Дата:
        !          2660: 25-Mar-2017
        !          2661: 
        !          2662: Редактор:
        !          2663: Александр Кудрявцев (Николаев)
        !          2664: 
        !          2665: Инфо:
        !          2666: Редактор благодарит за тестирование вопросов Тараса Вахрива, Дмитрия
        !          2667: Великова, Анастасию Гончарову, Екатерину Дубровскую, Андрея Кокуленко,
        !          2668: Юлию Лунёву, Максима Мерзлякова, Дениса Обуха, Дмитрия Овчарука, Алексея
        !          2669: Рабина, Аркадия Руха и Владимира Шлапака.
        !          2670: 
        !          2671: Вопрос 1:
        !          2672: (pic: 20160954.jpg)
        !          2673:    На раздаточном материале - рентгеновский ИКС, которым чаще других
        !          2674: пользовались девушки, имевшие ближневосточные или средиземноморские
        !          2675: корни. Девушек привлекало то, что эффект держался всю жизнь, а процедура
        !          2676: была абсолютно безболезненной. Назовите ИКС.
        !          2677: 
        !          2678: Ответ:
        !          2679: Эпилятор.
        !          2680: 
        !          2681: Зачет:
        !          2682: Депилятор.
        !          2683: 
        !          2684: Комментарий:
        !          2685: У девушек с ближневосточной или средиземноморской кровью нередко на лице
        !          2686: росли усики. С помощью рентгеновского облучения рост волос
        !          2687: останавливался. Правда, спустя какое-то время начинали проявляться
        !          2688: нежелательные последствия.
        !          2689: 
        !          2690: Источник:
        !          2691: http://www.cosmeticsandskin.com/cdc/xray.php
        !          2692: 
        !          2693: Автор:
        !          2694: Александр Кудрявцев (Николаев)
        !          2695: 
        !          2696: Вопрос 2:
        !          2697: После того как французский король Филипп I обозвал Вильгельма
        !          2698: Завоевателя бабой, последний некоторое время ограничивался только
        !          2699: спиртными напитками, надеясь исправить ситуацию. Какое слово с удвоенной
        !          2700: согласной мы пропустили в тексте вопроса?
        !          2701: 
        !          2702: Ответ:
        !          2703: Беременной.
        !          2704: 
        !          2705: Комментарий:
        !          2706: Вильгельм сильно растолстел и, после того как Филипп обозвал его
        !          2707: беременной бабой, решил похудеть, прибегнув к диете, которая состояла
        !          2708: исключительно из спиртных напитков.
        !          2709: 
        !          2710: Источник:
        !          2711: http://www.history.com/news/history-lists/10-things-you-may-not-know-about-william-the-conqueror
        !          2712: 
        !          2713: Автор:
        !          2714: Александр Кудрявцев (Николаев)
        !          2715: 
        !          2716: Вопрос 3:
        !          2717: Слово "ИКС" является заменой.
        !          2718:    Мать легендарного налетчика Джесси Джеймса тоже была не промах. После
        !          2719: смерти сына она организовала прибыльный бизнес, скупая старые ИКСЫ и
        !          2720: потом продавая их существенно дороже. Английский этимологический словарь
        !          2721: сообщает, что слово "ИКС" происходит от глагола со значением "свистеть".
        !          2722: Назовите ИКС.
        !          2723: 
        !          2724: Ответ:
        !          2725: Пистолет.
        !          2726: 
        !          2727: Комментарий:
        !          2728: Мать Джесси Джеймса обманывала покупателей, говоря, что все продаваемые
        !          2729: ею пистолеты принадлежали ее сыну. Английский этимологический словарь
        !          2730: указывает на происхождение слова "пистолет" от чешского
        !          2731: "pi&scaron;t'ala" [пистАла], которое, в свою очередь, происходит от
        !          2732: глагола "pisteti" [пистЕти] ("свистеть"). Однако если вы подумали, что
        !          2733: слово происходит от английского глагола "to whistle" [ту висл], - ничего
        !          2734: страшного.
        !          2735: 
        !          2736: Источник:
        !          2737:    1. http://en.wikipedia.org/wiki/Zerelda_James
        !          2738:    2. http://www.etymonline.com/index.php?term=pistol&allowed_in_frame=0
        !          2739: 
        !          2740: Автор:
        !          2741: Александр Кудрявцев (Николаев)
        !          2742: 
        !          2743: Вопрос 4:
        !          2744: Когда наивные телезрители стали интересоваться, как вырастить дерево у
        !          2745: себя дома, компания BBC [би-би-си] ответила, что саженец следует
        !          2746: посадить в жестянку с НИМ и надеяться на лучшее. Назовите ЕГО двумя
        !          2747: словами, которые начинаются на соседние буквы.
        !          2748: 
        !          2749: Ответ:
        !          2750: Томатный соус.
        !          2751: 
        !          2752: Комментарий:
        !          2753: В 1957 году в телепрограмме BBC "Панорама" в качестве первоапрельской
        !          2754: шутки был показан сюжет о необычайно богатом урожае макаронных деревьев
        !          2755: в Швейцарии.
        !          2756: 
        !          2757: Источник:
        !          2758: http://en.wikipedia.org/wiki/Spaghetti-tree_hoax
        !          2759: 
        !          2760: Автор:
        !          2761: Александр Кудрявцев (Николаев)
        !          2762: 
        !          2763: Вопрос 5:
        !          2764:    <раздатка>
        !          2765:    низм
        !          2766:    </раздатка>
        !          2767:    Существует вероучение, признаЮщее и уважающее все религии, в том
        !          2768: числе буддизм. Какие буквы мы пропустили в названии этого вероучения?
        !          2769: 
        !          2770: Ответ:
        !          2771: ом.
        !          2772: 
        !          2773: Комментарий:
        !          2774: Ом - широко употребляемая в буддизме мантра. "Omni" [Омни] - латинский
        !          2775: префикс, означающий "все" или "каждый".
        !          2776: 
        !          2777: Источник:
        !          2778:    1. http://en.wikipedia.org/wiki/Omnism
        !          2779:    2. http://en.wikipedia.org/wiki/Omni
        !          2780:    3. http://ru.wikipedia.org/wiki/Ом_(мантра)
        !          2781: 
        !          2782: Автор:
        !          2783: Александр Кудрявцев (Николаев)
        !          2784: 
        !          2785: Вопрос 6:
        !          2786: (pic: 20160955.jpg)
        !          2787:    Мадрид расположен в предгорьях массива СьЕрра-де-ГвадаррАма. В статье
        !          2788: об изображенной на раздаточном материале мадридской статуе сообщается и
        !          2789: ее высота над уровнем моря. Укажите эту высоту.
        !          2790: 
        !          2791: Ответ:
        !          2792: 666 метров.
        !          2793: 
        !          2794: Комментарий:
        !          2795: Это статуя Падшего Ангела в мадридском парке БуЭн-РетИро. Согласно
        !          2796: христианскому вероучению, падшие ангелы - это ангелы, взбунтовавшиеся
        !          2797: против Бога и прОклятые им за это. Первым и наиболее известным падшим
        !          2798: ангелом является Сатана.
        !          2799: 
        !          2800: Источник:
        !          2801: http://en.wikipedia.org/wiki/Fuente_del_%C3%81ngel_Ca%C3%ADdo
        !          2802: 
        !          2803: Автор:
        !          2804: Александр Кудрявцев (Николаев)
        !          2805: 
        !          2806: Вопрос 7:
        !          2807: (pic: 20160956.jpg)
        !          2808:    Через несколько часов после сооружения изображенная на снимке
        !          2809: пирамида была сожжена. Назовите двумя словами причину сожжения.
        !          2810: 
        !          2811: Ответ:
        !          2812: Сухой закон.
        !          2813: 
        !          2814: Зачет:
        !          2815: Запрет алкоголя.
        !          2816: 
        !          2817: Комментарий:
        !          2818: Пирамида сооружена из бочек с конфискованным алкоголем, которые через
        !          2819: несколько минут будут сожжены.
        !          2820: 
        !          2821: Источник:
        !          2822: http://www.kulturologia.ru/blogs/031115/27002/
        !          2823: 
        !          2824: Автор:
        !          2825: Александр Кудрявцев (Николаев)
        !          2826: 
        !          2827: Вопрос 8:
        !          2828: В прошлом веке в США группы добровольцев во избежание трагедий совершали
        !          2829: поисковые рейды по свалкам и повреждали ИХ, отрывая двери или выламывая
        !          2830: замки. Назовите ИХ.
        !          2831: 
        !          2832: Ответ:
        !          2833: Холодильники.
        !          2834: 
        !          2835: Комментарий:
        !          2836: В старых холодильниках замок не позволял открыть дверцу изнутри, и дети
        !          2837: нередко погибали, забравшись внутрь во время игр. Чтобы этого случайно
        !          2838: не произошло на свалке, добровольцы повреждали выброшенные холодильники.
        !          2839: 
        !          2840: Источник:
        !          2841: http://en.wikipedia.org/wiki/Refrigerator_death
        !          2842: 
        !          2843: Автор:
        !          2844: Александр Кудрявцев (Николаев)
        !          2845: 
        !          2846: Вопрос 9:
        !          2847: В XVII веке английские моряки стали кое-что делать на своем корабле в
        !          2848: знак скорби по погибшим товарищам, объясняя, что на время траура
        !          2849: освобождают место для невидимого ИКСА смерти. Так, по одной из версий,
        !          2850: появилась распространенная современная традиция. Назовите ИКС.
        !          2851: 
        !          2852: Ответ:
        !          2853: Флаг.
        !          2854: 
        !          2855: Комментарий:
        !          2856: Речь идет о традиции во время траура приспускать флаг.
        !          2857: 
        !          2858: Источник:
        !          2859: https://www.washingtonpost.com/news/morning-mix/wp/2015/07/22/a-sign-of-death-not-division-the-bloody-history-behind-lowering-flags-to-half-staff/
        !          2860: 
        !          2861: Автор:
        !          2862: Александр Кудрявцев (Николаев)
        !          2863: 
        !          2864: Вопрос 10:
        !          2865: В 30-е годы в СССР профессия летчика считалась верхом престижа. Поэтому
        !          2866: в одной телепрограмме утверждалось, что в указанное время мужчина в
        !          2867: лётной форме во всех смыслах являлся ИМ. Назовите ЕГО двукоренным
        !          2868: словом.
        !          2869: 
        !          2870: Ответ:
        !          2871: Небожитель.
        !          2872: 
        !          2873: Источник:
        !          2874: Хроники московского быта. Сын Кремля.
        !          2875: http://www.tvc.ru/channel/brand/id/37/show/episodes/episode_id/45326/
        !          2876: 
        !          2877: Автор:
        !          2878: Александр Кудрявцев (Николаев)
        !          2879: 
        !          2880: Вопрос 11:
        !          2881: Самолет, на котором летает американский президент, называют "Air Force
        !          2882: One" [эйр форс ван]. Прозвище самолета, на котором летает родившийся в
        !          2883: 2012 году Американский Фараон, двумя буквами отличается от названия
        !          2884: президентского самолета. Напишите это прозвище.
        !          2885: 
        !          2886: Ответ:
        !          2887: "Air Horse One" [чтецу: эйр хорс ван].
        !          2888: 
        !          2889: Комментарий:
        !          2890: Американский Фараон - кличка знаменитого скакуна. К местам, где проходят
        !          2891: скачки, его доставляют специальным самолетом.
        !          2892: 
        !          2893: Источник:
        !          2894:    1. http://www.eonline.com/news/663024/kentucky-derby-champion-american-pharoah-flies-on-a-plane-called-air-horse-one-yes-seriously
        !          2895:    2. http://en.wikipedia.org/wiki/American_Pharoah
        !          2896:    3. http://en.wikipedia.org/wiki/Air_Force_One
        !          2897: 
        !          2898: Автор:
        !          2899: Александр Кудрявцев (Николаев)
        !          2900: 
        !          2901: Вопрос 12:
        !          2902: (pic: 20160957.jpg)
        !          2903:    Поскольку фамилией ИКСА являлось старинное английское слово,
        !          2904: переводчик Владимир МЕдек использовал старочешское слово с тем же
        !          2905: значением, имевшее к тому же определенную фонетическую общность.
        !          2906: Назовите ИКСА.
        !          2907: 
        !          2908: Ответ:
        !          2909: [Альбус] Дамблдор.
        !          2910: 
        !          2911: Комментарий:
        !          2912: На раздаточном материале слово "шмель" написано на старочешском и
        !          2913: современном чешском языках. Фамилия Альбуса Дамблдора происходит от
        !          2914: староанглийского слова со значением "шмель"; сравните со словом
        !          2915: "bumblebee" [бАмблби]. Имя "Альбус" значит "белый", поэтому шмель на
        !          2916: раздаточном материале - белый.
        !          2917: 
        !          2918: Источник:
        !          2919: http://en.wikipedia.org/wiki/Harry_Potter_in_translation
        !          2920: 
        !          2921: Автор:
        !          2922: Александр Кудрявцев (Николаев)
        !          2923: 
        !          2924: Тур:
        !          2925: Финал (Минск). 3 тур
        !          2926: 
        !          2927: Дата:
        !          2928: 25-Mar-2017
        !          2929: 
        !          2930: Редактор:
        !          2931: Алексей Полевой (Гомель) и Денис Рыбачук (Брест)
        !          2932: 
        !          2933: Инфо:
        !          2934: Редакторы благодарят за помощь в работе над пакетом Максима Мерзлякова
        !          2935: (Воронеж), Сергея Терентьева и Бориса Моносова (оба - Санкт-Петербург),
        !          2936: Алексея и Марию Трефиловых (Калуга), Александра Кудрявцева (Николаев),
        !          2937: Николая Слюняева (Нижний Новгород), Наиля Фарукшина (Навои - Москва),
        !          2938: Дмитрия Капитанюка (Брест), Максима Новика (Гомель), Александра Огнева
        !          2939: (Краков), Арсэна Атнагулова (Уфа), Тараса Вахрива (Тернополь), Игоря
        !          2940: Тюнькина (Москва), Андрея Кокуленко (Омск).
        !          2941: 
        !          2942: Вопрос 1:
        !          2943: В русском переводе одного мультфильма гигант, насмехаясь над
        !          2944: противником, нараспев произносит обидное слово и повторяет его последний
        !          2945: слог. Напишите это слово.
        !          2946: 
        !          2947: Ответ:
        !          2948: Блоха.
        !          2949: 
        !          2950: Зачет:
        !          2951: Блоха-ха-ха и т.п.
        !          2952: 
        !          2953: Комментарий:
        !          2954: По сравнению с гигантом противник был очень мал. Переводчика этого
        !          2955: эпизода, видимо, вдохновила (пауза и ... если есть возможность, то
        !          2956: поставить музыкальный фрагмент отсюда (только звук, видео не надо) -
        !          2957: www.youtube.com/watch?v=LXELFcq5qJE с 46 секунды) песня Мефистофеля
        !          2958: композитора Мусоргского в исполнении Федора Шаляпина.
        !          2959: 
        !          2960: Источник:
        !          2961:    1. Мультфильм "Геркулес" (студия "Дисней", 1997).
        !          2962:    2. "Блоха" (Песня Мефистофеля в погребке Ауэрбаха), композитор М.П.
        !          2963: Мусоргский, слова И.В. Гёте, исполняет Ф.И. Шаляпин.
        !          2964: http://www.youtube.com/watch?v=LXELFcq5qJE
        !          2965: 
        !          2966: Автор:
        !          2967: Денис Рыбачук (Брест)
        !          2968: 
        !          2969: Вопрос 2:
        !          2970: В 1893 году иностранная фирма установила в одном из зданий российского
        !          2971: города лифт. Лев КЕкушев усовершенствовал его конструкцию и получил
        !          2972: премию. Что, согласно шутке современников, сделал с лифтом Кекушев?
        !          2973: 
        !          2974: Ответ:
        !          2975: Подковал [его].
        !          2976: 
        !          2977: Комментарий:
        !          2978: Похожая история произошла и с героем Лескова, подковавшим английскую
        !          2979: блоху. Фирма, правда, была не английская, а немецкая, город - не Тула, а
        !          2980: Москва, но современники шутили, что премия "за блоху, кою Лёва
        !          2981: подковал".
        !          2982: 
        !          2983: Источник:
        !          2984: http://moscow-i-ya.livejournal.com/368515.html
        !          2985: 
        !          2986: Автор:
        !          2987: Алексей Полевой (Гомель)
        !          2988: 
        !          2989: Вопрос 3:
        !          2990: В прошлые века при поиске залежей руды разведчики внимательно смотрели
        !          2991: под ноги во время поездки. Увиденная ОНА, по мнению Тимофея БажЕнова,
        !          2992: могла привести к открытию месторождения, рядом с которым запылают
        !          2993: плавильные печи. Назовите ЕЕ.
        !          2994: 
        !          2995: Ответ:
        !          2996: Искра.
        !          2997: 
        !          2998: Комментарий:
        !          2999: Разведчики смотрели под ноги лошади. Подкованное копыто в местах,
        !          3000: богатых металлическими рудами, могло высечь искру. Если месторождение
        !          3001: было богатым, то вскоре из искры могло возгореться пламя плавильных
        !          3002: печей. Стихотворение Пушкина с соответствующей строкой, кстати,
        !          3003: называется "Во глубине сибирских руд".
        !          3004: 
        !          3005: Источник:
        !          3006: Рейтинг Тимофея Баженова. Человек для опытов. Хозяйка медной горы.
        !          3007: https://russia.tv/video/show/brand_id/3879/episode_id/170455/
        !          3008: 
        !          3009: Автор:
        !          3010: Денис Рыбачук (Брест)
        !          3011: 
        !          3012: Вопрос 4:
        !          3013: Герои одного романа плохо разбираются в дореволюционных реалиях. Они
        !          3014: предполагают, что жившие в подвалах рабочие, чтобы выйти на солнечный
        !          3015: свет, должны были купить ИКС. Какие два слова мы заменили ИКСОМ?
        !          3016: 
        !          3017: Ответ:
        !          3018: Желтый билет.
        !          3019: 
        !          3020: Зачет:
        !          3021: Заменительный билет.
        !          3022: 
        !          3023: Комментарий:
        !          3024: Они думают, что билет желтый, так как по нему можно было выйти на
        !          3025: солнце. На самом деле желтый билет давал возможность легально заниматься
        !          3026: проституцией. Редакторы не исключают возможности того, что в зале есть
        !          3027: игроки, которые знали, что официальным названием "желтого билета" было
        !          3028: "заменительный билет", и ответили на вопрос, воспользовавшись
        !          3029: словом-подсказкой "заменили".
        !          3030: 
        !          3031: Источник:
        !          3032:    1. О.Д. Форш. Сумасшедший корабль. http://flibusta.is/b/268668/read
        !          3033:    2. http://ru.wikipedia.org/wiki/Жёлтый_билет
        !          3034: 
        !          3035: Автор:
        !          3036: Денис Рыбачук (Брест)
        !          3037: 
        !          3038: Вопрос 5:
        !          3039: В биографии Ильи Ильфа отмечается, что в первые дни после переезда в
        !          3040: Москву журналист искал для себя газету, отдавая предпочтение
        !          3041: широкоформатным. Это объяснялось отсутствием ЕГО. Какое произведение с
        !          3042: "НЕГО" начинается?
        !          3043: 
        !          3044: Ответ:
        !          3045: Мойдодыр.
        !          3046: 
        !          3047: Комментарий:
        !          3048: В то время Ильф был беден. Возможно, разыскивая газету пошире, чтобы
        !          3049: удобнее было укрываться, он повторял строчки недавно написанной
        !          3050: Чуковским сказки:
        !          3051:    Одеяло убежало,
        !          3052:    Улетела простыня,
        !          3053:    И подушка, как лягушка,
        !          3054:    Ускакала от меня.
        !          3055: 
        !          3056: Источник:
        !          3057:    1. Александра Ильф "Дом, милый дом".
        !          3058: http://tfile.co/forum/viewtopic.php?t=840279
        !          3059:    2. http://www.stihi-rus.ru/1/chukovskiy/14.htm
        !          3060: 
        !          3061: Автор:
        !          3062: Денис Рыбачук (Брест)
        !          3063: 
        !          3064: Вопрос 6:
        !          3065: Строительство началось во время Великой Депрессии, дало работу и помогло
        !          3066: выжить многим людям. Объект оказался очень дорогим - сооружение обошлось
        !          3067: в 35 миллионов долларов. Назовите этот объект.
        !          3068: 
        !          3069: Ответ:
        !          3070: Мост "Золотые ворота".
        !          3071: 
        !          3072: Комментарий:
        !          3073: На тот момент "Золотые ворота" оказались действительно "золотыми", за
        !          3074: период строительства начальная смета выросла вдвое. К сожалению, сейчас,
        !          3075: находясь в депрессии, многие используют мост совсем не по назначению.
        !          3076: 
        !          3077: Источник:
        !          3078:    1. http://gelio.livejournal.com/223154.html
        !          3079:    2. http://vse-chudesa.ru/chudesa-sveta-noveyshee-vremya/most-zolotie-vorota-v-san-francisko.html
        !          3080:    3. http://ru.wikipedia.org/wiki/Золотые_Ворота_(мост)
        !          3081: 
        !          3082: Автор:
        !          3083: Алексей Полевой (Гомель)
        !          3084: 
        !          3085: Вопрос 7:
        !          3086: В армии древних греков ИКСЫ одними из первых вступали в битву. Вскоре
        !          3087: после Берлинской олимпиады копия "ИКСА" оказалась в Германии. Какое
        !          3088: слово мы заменили ИКСОМ?
        !          3089: 
        !          3090: Ответ:
        !          3091: Дискобол.
        !          3092: 
        !          3093: Комментарий:
        !          3094: Диск использовался и как дальнее метательное оружие. Адольф Гитлер
        !          3095: считал статую "Дискобола" образцом для подражания и купил в Италии одну
        !          3096: из двух сохранившихся копий статуи Мирона. Кстати, статуя появляется в
        !          3097: начале фильма Лени Рифеншталь "Олимпия".
        !          3098: 
        !          3099: Источник:
        !          3100:    1. Документальный фильм "Происхождение Олимпийских игр" (Olympia -
        !          3101: The Origins of Games) (2016).
        !          3102: http://tfile.co/forum/viewtopic.php?t=888142
        !          3103:    2. http://www.bbc.com/russian/society/2015/04/150408_vert_cul_hitlers_idea_of_the_perfect_body
        !          3104: 
        !          3105: Автор:
        !          3106: Денис Рыбачук (Брест)
        !          3107: 
        !          3108: Вопрос 8:
        !          3109: Сотрудник американского журнала вспоминал, как в один прекрасный день
        !          3110: увидел ИКСА, который схватил АЛЬФУ быстро, как вспышка, и что обратил
        !          3111: внимание на контраст между ИКСОМ и АЛЬФОЙ. Назовите ИКСА и АЛЬФУ
        !          3112: словами, начинающимися на одну и ту же букву.
        !          3113: 
        !          3114: Ответ:
        !          3115: Матрос, медсестра.
        !          3116: 
        !          3117: Зачет:
        !          3118: Моряк, медсестра.
        !          3119: 
        !          3120: Комментарий:
        !          3121: (pic: 20160958.jpg)
        !          3122:    В один прекрасный день, когда американцы узнали об окончании войны,
        !          3123: они вышли на улицы, и начались гуляния. Фотограф Альфред Эйзенштадт
        !          3124: вспоминал, что матрос носился по улице быстро и целовал многих женщин,
        !          3125: но кадр с медсестрой получился особенно эффектным из-за контраста между
        !          3126: темной формой матроса и белым халатом медсестры. Дело было в Нью-Йорке,
        !          3127: а Нью-Йорк, как известно, - город контрастов.
        !          3128: 
        !          3129: Источник:
        !          3130: http://www.kp.by/daily/26401/3277924/
        !          3131: 
        !          3132: Автор:
        !          3133: Алексей Полевой (Гомель)
        !          3134: 
        !          3135: Вопрос 9:
        !          3136: Персонажа современного романа выгоняют из ТАКОГО клуба, после того как
        !          3137: тот использует банку с краской в качестве импровизированной ЕЕ. Назовите
        !          3138: ТАКУЮ ЕЕ.
        !          3139: 
        !          3140: Ответ:
        !          3141: Пейнтбольная граната.
        !          3142: 
        !          3143: Комментарий:
        !          3144: Персонаж использовал банку с краской как гранату, но в пейнтбольном
        !          3145: клубе его находчивость не оценили.
        !          3146: 
        !          3147: Источник:
        !          3148: Т. Пратчетт. Джонни и бомба. http://flibusta.is/b/486418/read
        !          3149: 
        !          3150: Автор:
        !          3151: Денис Рыбачук (Брест)
        !          3152: 
        !          3153: Вопрос 10:
        !          3154: КакАпо по кличке СирОкко нередко участвует в международных
        !          3155: природоохранных конференциях и выставках. По мнению создателей одного
        !          3156: документального фильма, СирОкко, в отличие от других какАпо, нельзя
        !          3157: назвать... Каким?
        !          3158: 
        !          3159: Ответ:
        !          3160: Нелетающим.
        !          3161: 
        !          3162: Комментарий:
        !          3163: Новозеландские какАпо - нелетающие птицы, как и киви, но Сирокко
        !          3164: является послом своего вида и символом охраны природы в Новой Зеландии,
        !          3165: поэтому часто летает на различные международные конференции, правда, на
        !          3166: самолетах.
        !          3167: 
        !          3168: Источник:
        !          3169: Документальный цикл "Дикая природа Новой Зеландии" (Wild New Zealand)
        !          3170: (2016), 3-я серия. http://tfile.co/forum/viewtopic.php?t=885320
        !          3171: 
        !          3172: Автор:
        !          3173: Денис Рыбачук (Брест)
        !          3174: 
        !          3175: Вопрос 11:
        !          3176: Яркие личинки жука мАйки, стремясь попасть в ИКС, собираются на кончиках
        !          3177: травинок. Назовите ИКС.
        !          3178: 
        !          3179: Ответ:
        !          3180: Улей.
        !          3181: 
        !          3182: Комментарий:
        !          3183: Нелетающие, но яркие личинки образуют подобие цветка и таким образом
        !          3184: приманивают пчел. Впоследствии они цепляются за пчелу и отпускают только
        !          3185: тогда, когда та прилетит в улей. Последствия для пчелиной семьи бывают
        !          3186: трагичными, так как личинки начинают поедать всё, что найдут в улье.
        !          3187: 
        !          3188: Источник:
        !          3189: Документальный цикл "Яд. Достижение эволюции" (2015), 3-я серия.
        !          3190: http://tfile.co/forum/viewtopic.php?t=878522
        !          3191: 
        !          3192: Автор:
        !          3193: Денис Рыбачук (Брест)
        !          3194: 
        !          3195: Вопрос 12:
        !          3196: В одном романе детей сначала приучают к взаимовыручке, а уже потом к
        !          3197: самостоятельности. Поэтому до определенного возраста глаза у детей
        !          3198: сзади, а потом - спереди. Какое слово в вопросе мы заменили?
        !          3199: 
        !          3200: Ответ:
        !          3201: Пуговицы.
        !          3202: 
        !          3203: Комментарий:
        !          3204: Когда все пуговицы на одежде сзади, то одеться ты можешь только с
        !          3205: чьей-либо помощью, а когда спереди, то уже и самостоятельно.
        !          3206: 
        !          3207: Источник:
        !          3208: Л. Лоури. Дающий. http://flibusta.is/b/236576/read
        !          3209: 
        !          3210: Автор:
        !          3211: Алексей Полевой (Гомель)
        !          3212: 
        !          3213: Тур:
        !          3214: Финал (Минск). 4 тур
        !          3215: 
        !          3216: Дата:
        !          3217: 25-Mar-2017
        !          3218: 
        !          3219: Редактор:
        !          3220: Серафим Шибанов (Москва), при участии Александра Карясова (Самара)
        !          3221: 
        !          3222: Инфо:
        !          3223: Сделать пакет лучше помогали: Анастасия Белова, Глеб Крутинин, Никита
        !          3224: Поздняков (все - Пущино), Виктория Бочкарёва, Сергей Кухарев, Дарья
        !          3225: Макушова, Юрий Мотькин, Регина Шарипова (все - Самара), Александр
        !          3226: Кудрявцев (Николаев), а также команды "Приматы" и "Слон потрогает тебя"
        !          3227: (обе - Самара).
        !          3228: 
        !          3229: Вопрос 1:
        !          3230: Компания "Alphabet" [Альфабет] перенесла свой сайт на новый домен,
        !          3231: который позиционируется как альтернатива .com [дот ком]. По мнению
        !          3232: автора вопроса, схожим образом в своем логотипе подчеркивает
        !          3233: разнообразие предоставляемых услуг и другая компания. Назовите эту
        !          3234: другую компанию.
        !          3235: 
        !          3236: Ответ:
        !          3237: "Amazon".
        !          3238: 
        !          3239: Комментарий:
        !          3240: Новый сайт "Alphabet" [Альфабет] находится по адресу abc.xyz [эй-би-си
        !          3241: дот икс-вай-зет], т.е., можно сказать, от A [эй] до Z [зет]. В логотипе
        !          3242: "AmaZon" [амазОн] от буквы A [эй] к букве Z [зет] идет стрелочка.
        !          3243: 
        !          3244: Источник:
        !          3245:    1. https://abc.xyz/
        !          3246:    2. http://en.wikipedia.org/wiki/Alphabet_Inc.
        !          3247:    3. https://www.amazon.co.uk/
        !          3248: 
        !          3249: Автор:
        !          3250: Илья Иванов (Путилково)
        !          3251: 
        !          3252: Вопрос 2:
        !          3253: "Гамбург" - единственный футбольный клуб, который играл во всех сезонах
        !          3254: немецкой Бундеслиги. За это клуб получил прозвище "ОН". Назовите фильм
        !          3255: 1993 года, в котором можно увидеть более десятка ИХ.
        !          3256: 
        !          3257: Ответ:
        !          3258: "Парк Юрского периода".
        !          3259: 
        !          3260: Комментарий:
        !          3261: Клуб прозвали Динозавром (иногда встречается вариант "Динозавр
        !          3262: Бундеслиги"). В фильме "Парк Юрского периода" много разных динозавров.
        !          3263: 
        !          3264: Источник:
        !          3265:    1. http://en.wikipedia.org/wiki/Hamburger_SV
        !          3266:    2. http://en.wikipedia.org/wiki/Jurassic_Park_(film)
        !          3267: 
        !          3268: Автор:
        !          3269: Серафим Шибанов (Москва)
        !          3270: 
        !          3271: Вопрос 3:
        !          3272: В песне группы "25/17" [двадцать пять семнадцать] отмечается, что
        !          3273: реальность и так достаточно сурова, и говорится, что бояться нужно не
        !          3274: ВТОРЫХ, а ПЕРВЫХ. Кто создал произведение, в заглавии которого
        !          3275: фигурируют ПЕРВЫЕ и ВТОРЫЕ?
        !          3276: 
        !          3277: Ответ:
        !          3278: [Константин] Симонов.
        !          3279: 
        !          3280: Комментарий:
        !          3281: Строчка из песни звучит так: "Все боятся мертвых, а нужно живых".
        !          3282: 
        !          3283: Источник:
        !          3284:    1. http://rusrap.org.ru/mp3/2517/text/Cherep_i_kosti.htm
        !          3285:    2. http://ru.wikipedia.org/wiki/Живые_и_мёртвые_(роман)
        !          3286: 
        !          3287: Автор:
        !          3288: Серафим Шибанов (Москва)
        !          3289: 
        !          3290: Вопрос 4:
        !          3291: Надеемся, этот вопрос будет вам приятен.
        !          3292:    В конце XIX века Лондон часто называли выражением, которое буквально
        !          3293: переводится как "Большой ОН". Согласно поговорке, ОН тесно связан... С
        !          3294: чем?
        !          3295: 
        !          3296: Ответ:
        !          3297: С огнем.
        !          3298: 
        !          3299: Комментарий:
        !          3300: Лондон известен своим смогом, поэтому получил прозвище Big Smoke [биг
        !          3301: смОук], что буквально переводится как "Большой дым". Согласно поговорке,
        !          3302: дыма без огня не бывает. В начале вопроса есть отсылка к цитате из "Горя
        !          3303: от ума" "И дым отечества нам сладок и приятен".
        !          3304: 
        !          3305: Источник:
        !          3306:    1. Дж.Э. Гарднер. Возвращение Мориарти.
        !          3307: http://flibusta.is/b/264025/read
        !          3308:    2. http://slovarick.ru/374/
        !          3309: 
        !          3310: Автор:
        !          3311: Серафим Шибанов (Москва)
        !          3312: 
        !          3313: Вопрос 5:
        !          3314: В своей книге Крис Тёрни поступил с погубившим древнюю сосну ученым так,
        !          3315: как должны были поступить с НИМ. В каком городе ОН жил?
        !          3316: 
        !          3317: Ответ:
        !          3318: Эфес.
        !          3319: 
        !          3320: Комментарий:
        !          3321: Крис Тёрни решил, что не стоит упоминать имя ученого, уничтожившего
        !          3322: столь ценный объект. Тот, кого древние греки запретили вспоминать, жил в
        !          3323: Эфесе и сжег местный храм Артемиды.
        !          3324: 
        !          3325: Источник:
        !          3326:    1. К. Тёрни. Кости, скалы и звезды. Наука о том, когда что произошло.
        !          3327: http://flibusta.is/b/320384/read
        !          3328:    2. http://en.wikipedia.org/wiki/Herostratus
        !          3329: 
        !          3330: Автор:
        !          3331: Игорь Тюнькин (Москва)
        !          3332: 
        !          3333: Вопрос 6:
        !          3334: Футболист МАриан ЧИшовски окончил карьеру в 2014 году. Незадолго до
        !          3335: завершения карьеры спортсмен принял участие в ЭТОМ с подачи хоккеиста
        !          3336: ВАцлава ПлЕтки. Назовите ЭТО тремя английскими словами.
        !          3337: 
        !          3338: Ответ:
        !          3339: Ice Bucket Challenge [чтецу: айс бАкет чЕллендж].
        !          3340: 
        !          3341: Зачет:
        !          3342: Айс бакет челлендж.
        !          3343: 
        !          3344: Комментарий:
        !          3345: Чишовски принял участие в флешмобе под названием "Ice Bucket Challenge"
        !          3346: [айс бакет чЕллендж], который должен был привлечь внимание к боковому
        !          3347: амиотрофическому склерозу. К сожалению, через пару месяцев после этого
        !          3348: Мариан узнал, что сам болен этой болезнью.
        !          3349: 
        !          3350: Источник:
        !          3351: http://www.sports.ru/tribuna/blogs/superratings/1182036.html
        !          3352: 
        !          3353: Автор:
        !          3354: Александр Карясов (Самара)
        !          3355: 
        !          3356: Вопрос 7:
        !          3357: [Ведущему: выделить голосом слово "распалась".]
        !          3358:    В 1918 году ЕМУ предложили должность в университете города Черновцы.
        !          3359: Пока ОН думал, ехать или не ехать, Австро-Венгрия распалась, и ОН
        !          3360: отказался. Назовите ЕГО.
        !          3361: 
        !          3362: Ответ:
        !          3363: [Эрвин] Шрёдингер.
        !          3364: 
        !          3365: Комментарий:
        !          3366: А если бы Австро-Венгрия не распалась, Шрёдингер принял бы это
        !          3367: предложение. В знаменитом мысленном эксперименте кот жив, если атом
        !          3368: радиоактивного вещества не распался, и мертв, если распался.
        !          3369: 
        !          3370: Источник:
        !          3371: http://ru.wikipedia.org/wiki/Шрёдингер,_Эрвин
        !          3372: 
        !          3373: Автор:
        !          3374: Александр Карясов (Самара)
        !          3375: 
        !          3376: Вопрос 8:
        !          3377:    <раздатка>
        !          3378:    В памятник Захер-Мазоху во Львове вмонтирована ОНА, через которую
        !          3379: можно разглядывать сменяющиеся эротические картинки. ONA изображена на
        !          3380: памятнике в европейской столице вместе... С кем?
        !          3381:    </раздатка>
        !          3382:    Вопрос перед вами. Время!
        !          3383: 
        !          3384: Ответ:
        !          3385: С Ромулом и Ремом.
        !          3386: 
        !          3387: Комментарий:
        !          3388: В первом случае имеется в виду лупа как прибор для улучшения зрения, а
        !          3389: во втором - Lupa capitolina - "Капитолийская волчица".
        !          3390: 
        !          3391: Источник:
        !          3392:    1. http://www.travel.ru/news/2008/03/24/121343.html
        !          3393:    2. http://it.wikipedia.org/wiki/Lupa_capitolina
        !          3394: 
        !          3395: Автор:
        !          3396: Александр Карясов (Самара)
        !          3397: 
        !          3398: Вопрос 9:
        !          3399: Советский ученый Владимир СперАнтов рассказывал, как его смелый знакомый
        !          3400: заявил, что устал от происходящего, и ушел к Сергею. Назовите фамилию
        !          3401: Сергея.
        !          3402: 
        !          3403: Ответ:
        !          3404: Прокофьев.
        !          3405: 
        !          3406: Комментарий:
        !          3407: Сергей Генкин, как и многие советские люди, присутствовал на похоронах
        !          3408: Сталина. Однако ему надоела толпа людей, и поэтому он захотел пойти на
        !          3409: похороны Прокофьева, который также умер 5 марта 1953 года.
        !          3410: 
        !          3411: Источник:
        !          3412: http://050353.ru/2016/03/04/sperantov/
        !          3413: 
        !          3414: Автор:
        !          3415: Александр Карясов, в редакции Юрия Мотькина (оба - Самара)
        !          3416: 
        !          3417: Вопрос 10:
        !          3418: ЕЕ дочерей зовут ОктАвия и СеверИна. В одном интервью ОНА призналась,
        !          3419: что в детстве была на домашнем обучении. Кто ОНА?
        !          3420: 
        !          3421: Ответ:
        !          3422: [Валерия Гай] Германика.
        !          3423: 
        !          3424: Комментарий:
        !          3425: Создательница сериала "Школа" сама в школу толком и не ходила.
        !          3426: 
        !          3427: Источник:
        !          3428:    1. http://ru.wikipedia.org/wiki/Германика,_Валерия_Гай_Александровна
        !          3429:    2. http://www.razgovorchiki.ru/arkhiv/germanika.htm
        !          3430: 
        !          3431: Автор:
        !          3432: Серафим Шибанов (Москва)
        !          3433: 
        !          3434: Вопрос 11:
        !          3435: В статье о НЕМ упоминается старовенгерский алфавит, придуманный
        !          3436: пастухами. Напишите слово, которое мы обозначили как ОН, при помощи
        !          3437: НЕГО.
        !          3438: 
        !          3439: Ответ:
        !          3440:    Бустро
        !          3441:    нодеф
        !          3442: 
        !          3443: Зачет:
        !          3444: Часть слова "бустрофедон" должна быть написана в одном направлении,
        !          3445: часть - в другом.
        !          3446: 
        !          3447: Комментарий:
        !          3448: Бустрофедон - способ письма, при котором чередуются строки, идущие слева
        !          3449: направо и справа налево.
        !          3450: 
        !          3451: Источник:
        !          3452: http://en.wikipedia.org/wiki/Boustrophedon
        !          3453: 
        !          3454: Автор:
        !          3455: Александр Карясов (Самара)
        !          3456: 
        !          3457: Вопрос 12:
        !          3458: Сериал "Во все тяжкие" повествует об Уолтере Уайте, которому
        !          3459: диагностируют неоперабельный рак. Вскоре после окончания сериала в
        !          3460: издании "Albuquerque Journal" [альбукЕрке джОрнал] появился ОН. Назовите
        !          3461: ЕГО точно.
        !          3462: 
        !          3463: Ответ:
        !          3464: Некролог [Уолтера] Уайта.
        !          3465: 
        !          3466: Комментарий:
        !          3467: Фанаты не поленились, и не важно, что Уолтер Уайт был всего лишь
        !          3468: персонаж.
        !          3469: 
        !          3470: Источник:
        !          3471: http://variety.com/2013/tv/news/breaking-bad-walter-white-obit-1200694265/
        !          3472: 
        !          3473: Автор:
        !          3474: Серафим Шибанов (Москва)
        !          3475: 
        !          3476: Тур:
        !          3477: Финал (Минск). 5 тур
        !          3478: 
        !          3479: Дата:
        !          3480: 25-Mar-2017
        !          3481: 
        !          3482: Редактор:
        !          3483: Мишель Матвеев (Санкт-Петербург)
        !          3484: 
        !          3485: Инфо:
        !          3486: Редактор благодарит тестеров: Антон Тахтаров, Яна Азриэль, Дмитрий
        !          3487: Овчарук, Наталья Орлова, Сергей Терентьев, Александр Коробейников,
        !          3488: Владимир Бройда, Александр Камаев, Юлия Фукельман, Сергей Лобачёв,
        !          3489: Владимир Городецкий, Ирина Зубкова, Антон Волосатов, Валерий Юдачёв,
        !          3490: Илья Чадаев, Андрей Данченко, Алексей Акименко, Глеб Олейник, Андрей
        !          3491: Кокуленко, Ваган Калайджян, Садиг Гамидов, Артем Рожков.
        !          3492: 
        !          3493: Вопрос 1:
        !          3494: Один человек был изуродован своими врагами, но в Петербурге нашелся
        !          3495: искусный специалист, который ему помог. Исследователь Крашенинников
        !          3496: усматривает в этом основу сюжета произведения. Какого?
        !          3497: 
        !          3498: Ответ:
        !          3499: "Нос".
        !          3500: 
        !          3501: Комментарий:
        !          3502: Упомянутому человеку враги отрезали нос, а петербургский мастер
        !          3503: изготовил для него протез. По мнению Крашенинникова, искусственный нос
        !          3504: мог навеять Гоголю фантазию о самостоятельных прогулках носа коллежского
        !          3505: асессора Ковалева.
        !          3506: 
        !          3507: Источник:
        !          3508: http://magazines.russ.ru/voplit/2001/5/kpas.html
        !          3509: 
        !          3510: Автор:
        !          3511: Мишель Матвеев (Санкт-Петербург)
        !          3512: 
        !          3513: Вопрос 2:
        !          3514: ОНИ сильно ограничивают свободу действий и поэтому, согласно Википедии,
        !          3515: стали символом женского целомудрия и мужской власти. Назовите процедуру,
        !          3516: результатом которой ОНИ становились.
        !          3517: 
        !          3518: Ответ:
        !          3519: Бинтование ног.
        !          3520: 
        !          3521: Зачет:
        !          3522: Забинтовывание ног; перебинтовывание ног; бинтовка ног.
        !          3523: 
        !          3524: Комментарий:
        !          3525: Женщина с ногами, изуродованными бинтованием, была ограничена в
        !          3526: возможности передвигаться самостоятельно, она была вынуждена сидеть дома
        !          3527: и не могла пойти куда-либо без сопровождения слуг.
        !          3528: 
        !          3529: Источник:
        !          3530: http://ru.wikipedia.org/wiki/Бинтование_ног
        !          3531: 
        !          3532: Автор:
        !          3533: Мишель Матвеев (Санкт-Петербург)
        !          3534: 
        !          3535: Вопрос 3:
        !          3536: Право посетить непосредственно сам ИКС получает очень небольшое
        !          3537: количество людей. В частности, один раз это могут сделать нобелевские
        !          3538: лауреаты. Назовите ИКС двумя словами.
        !          3539: 
        !          3540: Ответ:
        !          3541: Корабль "Ваза".
        !          3542: 
        !          3543: Зачет:
        !          3544: Корабль "Васа".
        !          3545: 
        !          3546: Комментарий:
        !          3547: Музей корабля "Ваза" в Стокгольме ежедневно посещает множество людей, но
        !          3548: экспозиция расположена вокруг корабля, а сам корабль берегут и
        !          3549: посетителей на него пускают лишь в исключительных случаях. В частности,
        !          3550: раз в году после вручения Нобелевской премии лауреаты получают право на
        !          3551: посещение.
        !          3552: 
        !          3553: Источник:
        !          3554:    1. Рассказ экскурсовода в музее "Ваза".
        !          3555:    2. http://cr2.livejournal.com/84353.html
        !          3556: 
        !          3557: Автор:
        !          3558: Мишель Матвеев (Санкт-Петербург)
        !          3559: 
        !          3560: Вопрос 4:
        !          3561: К герою Акунина, лежащему в реанимации после автокатастрофы, приходит
        !          3562: посетитель. Белье героя он называет словом женского рода. Каким?
        !          3563: 
        !          3564: Ответ:
        !          3565: Рубашка.
        !          3566: 
        !          3567: Зачет:
        !          3568: Сорочка.
        !          3569: 
        !          3570: Комментарий:
        !          3571: Идиома "родиться в рубашке" означает "быть везучим, счастливо избегать
        !          3572: беды". По мнению посетителя, белье героя, выжившего в автокатастрофе, -
        !          3573: это и есть та рубашка, в которой он родился.
        !          3574: 
        !          3575: Источник:
        !          3576: Б. Акунин. Фантастика. http://flibusta.is/b/182057/read
        !          3577: 
        !          3578: Автор:
        !          3579: Мишель Матвеев (Санкт-Петербург)
        !          3580: 
        !          3581: Вопрос 5:
        !          3582: На знаменитом мероприятии охрана может проверить любого посетителя,
        !          3583: выходящего из палатки. Так предотвращаются тысячи ИХ. Назовите ИХ тремя
        !          3584: словами.
        !          3585: 
        !          3586: Ответ:
        !          3587: Кражи пивных кружек.
        !          3588: 
        !          3589: Зачет:
        !          3590: Похищения пивных кружек; кражи/похищения пивных бокалов.
        !          3591: 
        !          3592: Комментарий:
        !          3593: Мероприятие - Октоберфест. Кружки крадут не только ради кружек, но еще и
        !          3594: просто потому, что это стало популярным развлечением. Приходится
        !          3595: принимать меры.
        !          3596: 
        !          3597: Источник:
        !          3598: http://ru.wikipedia.org/wiki/Октоберфест
        !          3599: 
        !          3600: Автор:
        !          3601: Мишель Матвеев (Санкт-Петербург)
        !          3602: 
        !          3603: Вопрос 6:
        !          3604: Микробиолог Блэкмор выяснил, что некоторые бактерии могут двигаться
        !          3605: необычным образом благодаря содержащимся в их организме кристаллическим
        !          3606: цепочкам железа. Аналогом чего являются эти цепочки?
        !          3607: 
        !          3608: Ответ:
        !          3609: Компаса.
        !          3610: 
        !          3611: Комментарий:
        !          3612: Эти бактерии могут двигаться строго на север или на юг, поскольку
        !          3613: упомянутые цепочки вытягиваются вдоль линий магнитного поля Земли.
        !          3614: 
        !          3615: Источник:
        !          3616:    1. С.И. Венецкий. В мире металлов. http://flibusta.is/b/224963/read
        !          3617:    2. http://ru.wikipedia.org/wiki/Магнетосома
        !          3618: 
        !          3619: Автор:
        !          3620: Мишель Матвеев (Санкт-Петербург)
        !          3621: 
        !          3622: Вопрос 7:
        !          3623: Сергей Бубка пошутил, что, когда в одном государстве происходит некое
        !          3624: событие, стоит такой рев, что его слышно в соседних государствах.
        !          3625: Назовите это событие.
        !          3626: 
        !          3627: Ответ:
        !          3628: Гран-при Монако.
        !          3629: 
        !          3630: Зачет:
        !          3631: Гонки "Формула-1" и т.п. по словам "Формула-1".
        !          3632: 
        !          3633: Комментарий:
        !          3634: Имеется в виду рев моторов. Монако - совсем маленький анклав внутри
        !          3635: Франции, а расстояние до итальянской границы составляет всего десяток
        !          3636: километров. Сергей Бубка, как и ряд других спортсменов, по окончании
        !          3637: карьеры поселился в Монако.
        !          3638: 
        !          3639: Источник:
        !          3640: https://esquire.ru/wil/bubka
        !          3641: 
        !          3642: Автор:
        !          3643: Мишель Матвеев (Санкт-Петербург)
        !          3644: 
        !          3645: Вопрос 8:
        !          3646: Дуплет.
        !          3647:    1. Персонаж одного романа называл ЕЕ Венерой. Назовите ЕЕ двумя
        !          3648: словами.
        !          3649:    2. Персонажу классического произведения в критический момент
        !          3650: показалось, что ОНА - это аристократка. Назовите ЕЕ двумя словами.
        !          3651: 
        !          3652: Ответ:
        !          3653:    1. Дама червей.
        !          3654:    2. Дама пик.
        !          3655: 
        !          3656: Комментарий:
        !          3657:    1. Венера - богиня любви, сердце - символ любви и символ червовой
        !          3658: масти.
        !          3659:    2. В "Пиковой даме" Германну в момент проигрыша померещилось, что
        !          3660: дама пик была старухой-графиней.
        !          3661: 
        !          3662: Источник:
        !          3663:    1. Б. Акунин. Азазель. http://flibusta.is/b/228606/read
        !          3664:    2. А.С. Пушкин. Пиковая дама.
        !          3665: http://www.rvb.ru/pushkin/01text/06prose/01prose/0866.htm
        !          3666: 
        !          3667: Автор:
        !          3668: Мишель Матвеев (Санкт-Петербург)
        !          3669: 
        !          3670: Вопрос 9:
        !          3671: Гравитон - это гипотетическая частица, переносящая гравитационное
        !          3672: взаимодействие. Часто задаются вопросом, почему ОН не оказывается
        !          3673: непреодолимым для гравитонов. Назовите ЕГО.
        !          3674: 
        !          3675: Ответ:
        !          3676: Горизонт событий [черной дыры].
        !          3677: 
        !          3678: Комментарий:
        !          3679: С одной стороны, черная дыра гравитационно взаимодействует с окружающим
        !          3680: миром. С другой стороны, согласно общей теории относительности, ничто, в
        !          3681: числе гравитон, не может выбраться из-за ее горизонта событий. Есть
        !          3682: несколько возможных способов разрешения этого противоречия - например,
        !          3683: гравитация может распространяться виртуальными гравитонами, рождающимися
        !          3684: вблизи горизонта событий. Окончательный ответ, видимо, зависит от того,
        !          3685: какой будет теория, в рамках которой будет описан гравитон.
        !          3686: 
        !          3687: Источник:
        !          3688:    1. https://www.quora.com/Can-gravitons-escape-the-event-horizon-of-a-black-hole-If-not-how-can-we-detect-the-black-holes-gravity
        !          3689:    2. http://physics.stackexchange.com/questions/107185/are-gravitons-bound-by-the-event-horizon
        !          3690:    3. http://curious.astro.cornell.edu/physics/89-the-universe/black-holes-and-quasars/theoretical-questions/451-how-do-gravitons-escape-black-holes-to-tell-the-universe-about-their-gravity-advanced
        !          3691:    4. https://www.google.ru/?gws_rd=ssl#q=graviton+event+horizon
        !          3692: 
        !          3693: Автор:
        !          3694: Мишель Матвеев (Санкт-Петербург)
        !          3695: 
        !          3696: Вопрос 10:
        !          3697: Согласно несерьезному мнению, это произведение следует признать
        !          3698: экстремистским за описание убийства главы государства при помощи дрона.
        !          3699: Назовите это произведение.
        !          3700: 
        !          3701: Ответ:
        !          3702: "Сказка о золотом петушке".
        !          3703: 
        !          3704: Комментарий:
        !          3705: Золотой петушок, убивший царя Додона, - вполне себе дрон. А домик Элли,
        !          3706: если что, не был управляемым, да и Гингема - не глава государства.
        !          3707: 
        !          3708: Источник:
        !          3709: http://www.anekdot.ru/id/822764/
        !          3710: 
        !          3711: Автор:
        !          3712: Мишель Матвеев (Санкт-Петербург)
        !          3713: 
        !          3714: Вопрос 11:
        !          3715: Одна из версий объясняет происхождение известного спортивного термина
        !          3716: так: он образован от французского слова "l'&oelig;uf" [лёф], означающего
        !          3717: яйцо, а яйцо похоже на... Что?
        !          3718: 
        !          3719: Ответ:
        !          3720: Ноль.
        !          3721: 
        !          3722: Комментарий:
        !          3723: В теннисе ноль очков почему-то называют словом "love" [лав]. Причины и
        !          3724: пытается объяснить приведенная версия.
        !          3725: 
        !          3726: Источник:
        !          3727: http://en.wikipedia.org/wiki/Tennis_scoring_system
        !          3728: 
        !          3729: Автор:
        !          3730: Мишель Матвеев (Санкт-Петербург)
        !          3731: 
        !          3732: Вопрос 12:
        !          3733: В рассказе об известном произведении говорится, что в Карлсбаде некто
        !          3734: пошутил: "Видите, как действуют на организм карлсбадские воды!".
        !          3735: Назовите это произведение.
        !          3736: 
        !          3737: Ответ:
        !          3738: "Прощальная симфония".
        !          3739: 
        !          3740: Комментарий:
        !          3741: По версии шутника, музыканты один за другим уходили со сцены по личным
        !          3742: мотивам. Кстати, исполнялась не сама "Прощальная симфония" Гайдна, а
        !          3743: произведение Рихарда Штрауса, использовавшее ту же идею постепенного
        !          3744: ухода музыкантов.
        !          3745: 
        !          3746: Источник:
        !          3747: http://www.blagaya.ru/put/articles/muzhumor/
        !          3748: 
        !          3749: Автор:
        !          3750: Мишель Матвеев (Санкт-Петербург)
        !          3751: 

FreeBSD-CVSweb <freebsd-cvsweb@FreeBSD.org>